SlideShare a Scribd company logo
1 of 20
1. Secretion of what gastrointestinal              C. 0%
hormones will be primarily decreased as a          D. 100%
result of iduodenum removal?                       E. 75%
A. Histamine                                       7. Scraps of the mycelium of a fungus, spores,
B. Gastrin and histamine                           air bubbles and fat drops were discovered
C. Cholecystokinin and secretin                    on microscopy of the patient's hair excluded
D. Gastrin                                         from the infected areas. What fungus disease
E. Neurotensin                                     is characterised by this microscopic picture?
2. Low level of albumins and fibrinogen was        A. Sporotrichosis
detected in the patient's blood. Decreased         B. Favus
activity of what organelle of the liver            C. Epidermophytosis
hepatocytes can most probably cause it?            D. Trichophytosis
A. Granular endoplasmatic reticulum                E. Microspory
B. Lysosomes                                       8. From the nasopharynx of a 5-year-old child
C. Golgi complex                                   it was excreted a microorganism which
D. Mitochondrions                                  is identical to Corynebacterium diphtheriae
E. Agranular endoplasmatic reticulum               dose according to morphological and biochemical
3. A 10-year-old girl often experiences acute      signs. But this microorganism
respiratory infections with multiple punctate      does not produce exotoxin. As a result
haemorrages in the places of clothes friction.     of what process can this microorganism
Hypovitaminosis of what vitamin has the            become toxigenic?
girl?                                              A. Growing with antitoxic serum
A. B2                                              B. Phage conversion
B. C                                               C. Chromosome mutation
C. B6                                              D. Passing through the organism of the
D. B1                                              sensitive animals
E. A                                               E. Cultivation in the telluric environment
4. Different functional groups can be              9. A patient has undergone an amputation
presented in the structure of L-amino acid's       of lower extremity. Some time later painful
radicals. Identify the group that is able to       nodules appeared in a slump. Amputations
form ester bond:                                   neuromas were found out at the microscopic
A. -OH                                             examination. To what pathological processes
B. -NH2                                            do those formations relate?
C. -CONH2                                          A. Inflammation
D. -CH3                                            B. Regeneration
E. -SH                                             C. Hyperemia
5. A 27-year-old woman has dropped penicillin      D. Metaplasia
containing eye drops. In a few minutes             E. Dystrophy
there appeared feeling of itching, burning         10. To prevent long-term effects of 4-day
of the skin, lips and eyelids edema, whistling     malaria a 42-year-old patient was prescribed
cough, decrease of BP. What imunoglobulins         primaquine. On the 3-rd day from the
take part in the development of this allergic      begin of treatment there appeared stomach
reaction?                                          and heart pains, dyspepsia, general cyanosis,
A. IgM and IgG                                     hemoglobinuria. What caused side effects of
B. lgA and IgM                                     the preparation?
C. IgM and IgD                                     A. Drug potentiation by other preparations
D. IgG and IgD                                     B. Cumulation of the preparation
E. IgE and IgG                                     C. Delayed urinary excretion of the
6. Hypertrychosis of auricles is caused by         preparation
a gene that is localized in Y-chromosome.          D. Decreased activity of microsomal liver
Father has this feature. What is the probability   enzymes
to give birth to a boy with such anomaly?          E. Genetic insufficiency of glucose 6-phosphate
A. 25%                                             dehydrogenase
B. 35 %                                            11. A patient with diabetes mellitus experienced
loss of consciousness and convulsions after       D. Balantidium
injection of insulin, what is the                 E. Intestinal amoeba
result of biochemical blood analysis for          16. A 44-year--old woman complains of
concentration of the sugar?                       common weakness, heart pain., considerable
A. 3,3 mmol/L                                     increase of body weigt. Objectively: moonlike
B. 8,0 mmol/L                                     face, hirsutism, AP- 165/100 mm Hg.
C. 5,5 mmol/L                                     height- 164 cm, weight - 103 kg: fat is mostly
D. 1,5 mmol/L                                     accumulated in the region of neck, upper
E. 10,0 mmol/L                                    shoulder girdle, stomach. What is the main
12. A patient with complaints of dryness in       pathogenetic mechanism of obesity?
the mouth, photophobia and vision impairment      A. Decreased production of thyroidal
was admitted in the reception-room.               hormones
Skin is hyperemic, dry, pupils, are dilated,      B. Increased production of insulin
tachycardia. Poisoning with belladonna            C. Decreased production of glucagon
alkaloids was diagnosed on further examination.   D. Increased production of glucocorticoids
What medicine should be prescribed?               E. Increased production of mineralocorticoids.
A. Armine                                         17. Inflammatory process or modified
B. Dipiroxim                                      subserous layer around cervix or the uterus
C. Pilocarpine                                    caused an intensive pain syndrome. In what
D. Diazepam                                       region of genitals does the pathological
E. Prozerin                                       process take place?
13. A patient suffering from trombophlebitis of   A. Endometrium
the deep crural veins suddenly died.              B. Perimetrium
Autopsy has shown freely lying red                C. Mesometrium
friable masses with dim crirnped surface          D. Parametrium
In the trunk and bifurcation of the               E. Myometrium
pulmonary artery. What pathologic process         18. Microspecimen of red bone marrow
was revealed by morbid anatomist?                 contains multiple capillares through the
Ac. Thrombosis                                    walls of which mature blood cells penetrate
B. Tromboembolism                                 into the bloodstream. What type of capillares
C. Tissue embolism                                are these?
D. Fat embolism                                   A. Fenestrational
E. Embolism with foreign body?                    B. Sinusoidal
14. A 38-year-old man who poisoned himself        C. Somatical
with mercury dichloride was taken to the          D. Visceral
admission room in grave condition. What           E. Lymphatic
antidote should be immediately introduced?        19. A patient with thrombophlebitis of
A. Isonitrosine                                   lower extremities had got chest pains, blood
B. Dipiroxim                                      spitting, growing respiratory failure that
C. Unithiol                                       caused his death. Autopsy revealed multiple
D. Nalorphine                                     pulmonary infarctions. What is the most
E. Atropine                                       probable reason of their development?
15. Slime, blood and protozoa 30-200 microns      A. Pulmonary artery thrombosis
long have been revealed in a man's                B. Bronchial artery thrombosis
feces. The body is covered with cilias and has    C. Bronchial artery embolism
correct oval form with a little bit narrowed      D. Pulmonary venous thrombosis
anterior and wide round shaped posterior          E. Pulmonary artery embolism
end. At the anterior end a mouth is visible.      20. An isolated cell of human heart
In cytoplasm there are two nucleui and            automatically generates excitation impulses
two short vacuoles. What are the described        with frequency 60 times pro minute. What
features typical for?                             structure does this cell belong to?
A. Lamblia                                        A. Ventricle
B. Dysenteric amoeba                              B. Sinoatrial node
C. Trichomonas                                    C. Atrioventricular node
D. His' bundle                                      B. Satellite-cells
E. Atrium                                           C. Fibroblasts
21. During examination of a patient, there          D. Myofibroblasts
was found a neoplasm in the white substance         E. Myoblasts
of cerebral hemispheres with localization in the    26. An autopsy revealed large (1-2 cm)
knee and frontal part of posterior crus of          brownish-red, easy crumbling formations
internal capsule. Fibres of what conductive         covering ulcerative defects on the external
tract of' the brain will be disrupted?              surface of the aortic valve. That is the most
A. Tr. frontothalamicus                             likely diagnosis?
B. Tr. parietooccipitopontinus                      A. Diffusive endocarditis
C. Tr. pyramidalis                                  B. Fibroplastic endocarditis
D. Tr. frontopontinus                               C. Acute verrucous endocarditis
E. Tr. thalamocorticalis                            D. Polypous-ulcerative endocarditis
22. A sick man with high temperature and a          E. Recurrent warty endocarditis
lot of tiny wounds on the body has been             27. A patient died from acute cardiac
admitted to the hospital. Lice have been            insufficiency. The histological examination
found in the folds of his clothing. What disease    of his heart revealed the necrotized section
can be suspected?                                   in myocardium of the left ventricle, which
A. Plague                                           was separated from undamaged tissue
B. Epidemic typhus                                  by the zone of hyperimic vessels, small
C. Malaria                                          hemorrhages and leukocytic infiltration.
D. Tularemia                                        What is the most likely diagnosis?
E. Scabies                                          A. Diffuse exudate myocarditis
23. Autopsy of a 46-year-old man revealed           B. Productive myocarditis
Multiple brown-and-green layers and                 C. Focal exudate myocarditis
hemmorhages on the mucous membrane                  D. Myocardial ischemic dystrophy
of rectum and sigmoid colon; slime and              E. Myocardial infarction
some blood in colon lumen; histologically           28. A patient operated on complicated
- fibrinous colitis. In course of bacteriological   appendicitis has the following changes of
analysis of colon contents S.Sonne were             blood count: erythrocytes – 4,0 ·1012 /1,
found. What is the most probable diagnosis?         Hb - 120 gll. color index - 0.9, leukocytes
A. Crohn’s disease                                  - 18·109 /1, basophils - 0, eosinophils - 0,
B. Cholera                                          myelocytes - 0, juvenile - 0, stab neutrophils
C. Salmonellosis                                    - 20, segmentonuclear neutrophils - 53,
D. Dysentery                                        lymphocytes - 21, monocytes - 5. How is such
E. Yersiniosis                                      nuclear shift of leukocy1ic formula called?
24. Intake of oral contraceptives containing        A. Degenerative left shift
sex hormones inhibits secretion of the              B. Regeneratively- degenerative
hypophysiae hormones. Secretion of which            C. Hyperregenerative
of the indicated hormones is inhibited              D. Right shift
while taking oral contraceptives with sex           E. Regenerative left shift
hormones?                                           29. A tissue sample of benign tumor was
A. Somatotropic                                     studied under the electron microscope. A lot
B. Oxytocin                                         of small (15-20 nm) spherical bodies, consisting
C. Vasopressin                                      of 2 unequal subunits were detected.
D. Follicle-stimulating                             These are:
E. Thyrotropic                                      A. Mitochondria
25. Patient with injured muscles of the             B. Smooth endoplasmic reticulum
lower extremities was admitted to the               C. Ribosomes
traumatological department. Due to what             D. Golgi complex
cells is reparative regeneration of the muscle      E. Microtubules
fibers and restoration of the muscle function       30. A 45-year-old man applied to the
possible?                                           trauma station because of domestic shoulder
A. Myoepithelial cells                              trauma. Objectively: extension, reduction
and pronation functions of the shoulder are       dystrophic changes of parenchymatous
absent. What muscle was injured?                  organs, brain edema. What complication can
A. Supraspinous muscle                            cause such changes of serous membranes
B. Subscapular muscle                             and inner organs?
C. Infraspinous muscle                            A. Thrombopenia
D. Teres minor muscle                             B. DIC-syndrome
E. Teres major muscle                             C. Anemia
31. Kidneys of a man under examination            D. Uraemia
show increased resorbtion of calcium ions and     E. Sepsis
decreased resorbtion of phosphate ions.           36. In course of prophylactic medical
What hormone causes this phenomenon?              examination 7-year-old boy was
A. Parathormone                                    diagnosed to have daltonism.
B. Hormonal form D3                                Parents are healthy color
C. Aldosterone                                    vision is normal. But grandfather from the
D. Thyrocalcitonin                                mother’s side has the same disorder.
E. Vasopressin                                    What is the type of inheriting of this anomaly?
32. Increased fragility of vessels, enamel and    A. Incomplete domination
dentine destruction resulting from scurvy         B. Autosomal - dominant
are caused by disorder of collagen maturation.    C. Recessive, sex-linked
What stage of procollagen modification            D. Dominant, sex-linked
is disturbed under this avitaminosis?             E. Autosomal - recessive
A. Hydroxyla1ion of proline                       37. A patient with encephalopathy
B. Formation of polypeptide chains                was admitted to the neurological inpatient
C. Detaching of N-ended peptide                   department. There was revealed
D. Removal of C-ended peptide from                a correlation between increasing of
procollagen                                       encephalopathy and substances absorbed by
E. Glycosylation of hydroxylysine residues        the bloodstream from the intestines. What
33. A 7-year-old girl has signs of anemia.        substances that are formed in the intestines
Laboratory examination revealed pyruvate          can cause endotoxemia?
kinase deficiency in erythrocytes. What           A. Biotin
process disturbance plays· the main role in       B. Indole
anemia deve1opment?                               C. Acetacetate
A. Oxidative phosphorylation                      D. Butyrate
B. Anaerobic glycolysis                           E. Ornithine
C. Tissue respiration                             38. A patient with hypochromic anemia has
D. Peroxide decomposition                         splitting hair and loss of hair, increased nail
E. Aminoacids desamination                        brittling and taste alteration. What is the
34. During the endoscopy the inflammation         mechanism of the development of these
of a major papilla of the duodenum and the        symptoms?
disturbances of bile secretion were found. In     A. Decreased production of thyroid
which part of duodenum were the problems          hormones
found?                                            B. Deficiency of vitamin A
A. Upper horizontal part                          C. Decreased production of parathyrin
B. Ascendant part                                 D. Deficiency of vitamin B12
C. Bulb                                           E. Deficiency of iron-containing enzymes
D. Lower horizontal part                          39. A patient visited a dentist with complaints
E. Descendent part                                of redness and edema of his mouth
35. For a long time a 49-year-old woman           mucous membrane in a month after dental
had suffered from glomerulonephritis which        prosthesis. The patient was diagnosed with
caused death. The autopsy revealed that the       allergic stornatitis. What type of allergic
size of her kidney's was 7 x 3 x 2,5 cm, weight   reaction by Gell and Cumbs underlies this
65, 0 g, they were dense and small-grained.       disease?
Microscopically fibrinogenous inflammlation       A. Cytotoxic
of serous and mucous membranes,                   B. Delayed type hypersensitivity
C. Stimulating                                       permeability of vessels of microcirculation
D. Immunocomplex                                     stream, increase of their hydrodynamic
E. Anaphylactic                                      blood pressure. Increasing of the osmotic
40. Most participants of Magellan expedition         concentration and dispersity of protein
to America died from avitominosis. This              structures is present in the intercellular fluid.
disease declared itself by general weakness,         What kind of edema will appear in this case?
subcutaneous hemorrhages, falling of                 A. Colloid-osmotic
teeth, gingival hemorrhages. What is the             B. Lymphogenic
name of this avitominosis?                           C. Mixed
A. Polyneuritis (beriberi)                           D. Hydrodynamic
B. Pellagra                                          E. Membranogenic
C. Rachitis                                          46. A 46-year-old man complains of difficulties
D. Scurvy                                            with nasal breathing. Mikulicz's
E. Biermer's anemia                                  cells, accumulation of epithelioid cells,
41. A 17-year-old boy fell seriously ill body        plasmocytes, lymphocytes; hyaline balls
temperature rose up to 38,5°C, there                 were discovered in the biopsy material of the
is cough, rhinitis, lacrimation, nasal discharges.   thickened nasal mucosa. What is the most likely
What kind of inflammation is it?                     diagnosis?
A. Suppurative inflammation                          A. Meningococcal nasopharyngitis
B. Hemorrhagic inflammation                          B. Rhinovirus infection
C. Serous inflammation                               C. Virus rhinitis
D. Fibrinous inflammation                            D. Scleroma
E. Catarrhal inflammation                            E. Allergic rhinitis
42. The donor who didn't donate the blood            47. A 60-year-old patient fell ill with
for a long time was investigated with                obturative jaundice as a result of malignant
IFA method. Anti-HBs antibodies were                 tumour of the big papillary of the duodenal.
revealed. What does positive result of IFA           Lumen of what anatomical structure is
in this case mean?                                   compressed by tumour?
A. Previous hepatitis B                              A. Left hepatic duct
B. Chronic hepatitis C                               B. Common hepatic duct
C. Chronic hepatitis B                               C. Right hepatic duct
D. Acute hepatitis B                                 D. Hepatopancreatic ampulla
E. Acute hepatitis C                                 E. Cystic duct
43. A 42-year-old man suffering from gout            48. Desulfiram is widely used in medical
has increased level of urinary acid in blood.        practice to prevent alcocholism. It inhibits
Allopurinol was prescribed to decrease the           aldehyde dehydrogenase. Increased
level of urinary acid. Competitive inhibitor         level of what metabolite causes aversion to
of what enzyme is allopurinol?                       alcochol?
A.Xanthine oxidase                                   A. Propionic aldehyde
B. Guanine deaminase                                 B. Methanol
C. Adenosine deaminase                               D. Ethanol
D. Hypoxanthine phosphoribosiltransferase            D. Acetaldehyde
E. Adenine phosphoribosiltransferase                 E. Malonyl aldehyde
44. While preparing a patient to the operation       49. An experiment proved that UV-radiated
the heart chambers' pressure was                     cells of patients with xeroderma
measured. In one of them the pressure                pigmentosum restore the native DNA
changed during one heart cycle from 0 to             structure slower than cells of healthy individuals
120 mm Hg. What chamber of heart was it?             as a result of reparation enzyme
A. Left atrium                                       defection. What enzyme helps this process?
B. Left ventricle                                    A. Primase
C. Right ventricle                                   B. DNA polymerase III
D. -                                                 C. DNA gyirase
E. Right atrium                                      D. RNA ligase
45. Inflammation is characterised bv increased       E. Endonuclease
50. An old woman was hospitalized with           A. NK-cells
acute pain, edema in the right hip joint; the    B. Macrophages, monocytes
movements in the joint are limited. Which        C. 0-lymphocytes
bone or part of it was broken?                   D. T-lymphocytes, B-lymphocytes
A. The body of the thigh bone                    E. Fibroblasts, T-lymphocytes, B-lymphocytes
B. Ischial bone                                  55. The effect of electric current on the
C. Pubic bone                                    exitable cell caused depolarization of its
D. Condyle of the thigh                          membrane. Movement of what ions through
E. The neck of the thigh                         the membrane caused depolarization?
51. In the blood of a 26-year-old man            A. HCO3-
18% of erythrocytes of the spherical,            B. Ca2+
ballshaped, flat and spinous shape have been     C. K+
revealed. Other erythrocytes were in form        D. Cl-
of the concavo-concave disks. How is this        E. Na+
phenomenon called?                               56. A patient with diabetes mellitus has been
A. Pathological anisocytosis                     delivered in hospital in the state of
B. Pathological poikilocytosis                   unconsciousness. Arterial pressure is low.
C. Erytrocytosis                                 The patient has acidosis. Point substances,
D. Physiological poikilocytosis                  which accumulation in the blood results in these
E. Physiological anisocytosis                    manifestations:
52. Testosterone and it’s analogs increase       A. High fatty acids
the mass of skeletal muscles that allows to      B. Amino acids
use them for treatment of dystrophy. Due to      C. Ketone bodies
interaction of the hormone with what cell        D. Cholesterol esters
substrate is this action caused?                 E. Monosaccharides
A. Membrane receptors                            57. Synthesis of phospholipids is disturbed
B. Chromatin                                     as a result fatty infiltration of liver. Indicate
C. Ribosomes                                     which of the following substances can
D. Nuclear receptors                             enhance the process of methylation during
E. Proteins-activators of transcription          phospholipids synthesis?
53. A 39-vear-old woman has madescence in        A. Ascorbic acid
the region or mammilla, a small ulcer with       B. Glucose
inflammatory hyperemia and cutaneous             C. Methionine
edema. Histologic examination of tissue          D. Glycerin
sampling from this area revealed in the          E. Citrate
malpighian layer of thickened epidermis          58. Donor skin transplantation was
atypical cells with light and optically empty    performed to a patient with extensive burns.
cytoplasm. With no intracellular bridges.        On the 8-th day the graft became swollen
Such cells were also found in the orifice of     and changed color; on the 11-th day graft
big mammal gland ductus. What is the most        rejection started. What cells take part in this
probable diagnosis?                              process?
A. Epidermoid cancer                             A. B-lymphocytes
B. Melanocarcinoma                               B. Erythrocytes
C. Basal cell carcinoma                          C. Basophils
D. Intraductal cancer                            D. T-lymphocytes -
E. Paget’s disease                               E. Eosinophils
54. A patient with clinical signs of             59. A 38-year-old patient died during
immunodeficiency has no changes of               intractable attack of bronchial asthma.
the number and functional activity of            Histologic examination revealed mucus
T- and B- lymphocytes. Defect with dysfunction   accumulation in bronchial lumen, a lot of
of antigen-presentation to the                   fat cells (labrocytes) in the wall of bronches,
immunocompetent cells was found during           many of them are in the state of degranulation,
examinatio on the molecule level. Defect of      there are also a lot of eosinophils. What
what cells is the most probable?                 pathogenesis of bronchial changes is it?
A. Granulomatosis                                C. Pus in the wound
B. Immunocomplex mechanism                       D. Low concentration H2O2
C. Cellular cytolysis                            E. Inherited insufficiency of catalase
D. Atopy                                         65. A patient with continious bronchopneumonia
E. Cytotoxic, cytolytic action of antibodies     was admitted to the therapeutic department.
60. A 2-year-old child has got intestinal        Antibiotic therapy didn't give much effect.
dysbacteriosis, which resultsed in               What medication for improvement of
hemorrhagic syndrome. What is the most           immune state should be added to the
likely cause of hemorrhage of the child?         complex treatment of this patient?
A. Fibrinogen deficiency                         A. Timaline
B. Hypocalcemia                                  B. Analgin
C. PP hypovitaminosis                            C. Sulfocamphocaine
D. Vitamin K deficiency                          D. Paracetamol
E. Activation of tissue thromboplastin           E. Benadryl
61. Autopsy revealed that right lung is          66. Decreased blood supply to the organs
enlarged, solid, there are fibrin layers on      causes hypoxia that activates fibroblasts
the pleura. Lung tissue is light grey color on   function. Volume of what elements is
incision with muddy liqued exudates.What         increased in this case?
lung disease are these symptoms typical for?     A. Parenchymatous elements of an organ
A. Interstitial pneumonia                        B. Vessels of microcircular stream
B. Bronchopneumonia                              C. Lymphatic vessels
C. Croupous pneumonia                            D. Intercellular substance
D. Pulmonary gangrene                            E. Nerve elements
E. Fibrosing alveolilis                          67. A doctor administered Allopurinol to a
62. Blood analysis of a patient showed signs     26-year-old young man with the symptoms
of HIV infection (human immunodeficiency         of gout. What pharmacological action of
virus). Which cells does HIV-virus primarily     Allopurinol ensures therapeutical effect?
affect?                                          A. By inhibiting leucocyte migration into the joint
A. Proliferating cells (stem hematoplastic       B. By general analgetic effect
cells)                                           C. By increasing uric acid excretion
B. Specialized nervous cells (neurons)           D. By general anti-inflammatory effect
C. Cells that contain receptor IgM               E. By inhibiting uric acid synthesis
(B-lymphocytes)                                  68. Thyrotoxicosis leads to increased
D. Mast cells                                    production of thyroidal hormones T3 and
E. Cells that contain receptor T4 (T-helpers)    T1, weight loss, tachycardia, psychic
63. A 60-year-old patient has reduced            excitementand so on. How do thyroidal
perception of high-frequency sounds. What        hormones effect energy metabolism in the
structures' disorder of auditory analyzer        mitochondrion of cells?
caused these changes?                            A. Disconnect oxidation and oxidative
A. Eustachian tube                               phosphorylation
B. Main membrane of cochlea near the oval        B. Stop respiratory chain
window                                           C. Activate oxidative phosphorylation
C. Muscles of middle car                         D. Stop substrate phosphorylation.
D. Main membrane of cochlea near helicotrerna    E. Activate substrate phosphorylation
E. Tympanic membrane                             69. A patient with rheumatoid arthritis who
64. Patient with abscess of the cut wound        had been treated with indometacin has got
applied to the traumatological department.       signs of gastropathy.thy. What activity of the
In order to clean the wound from the pus         drug can this complication be connected with?
doctor washed it with 3% hydrogen peroxide.      A. Anticyclooxygenase
Foam was absent. What caused the                 B. Antiserotonin
absence of the drug activity?                    C. Antihistamine
A. Shallow wound                                 D. Locally irritating
R. Inherited insufficiency of erythrocyte        E. Antikinine
phosphatdehydrogenase.                           70. Dystrophic alterations of heart are
accompanied with dilation of heart cavities,     A. Parathormone
decreased force of heart contractions,           B. Aldosterone
increased blood volume that remains during       C. Adrenalin
systole in the heart cavity, vein overfill.      D. Vasopressin
What heart condition is it typiccal for?         E. Auricular sodiumuretic factor
A. Myogenic dilatation                           76. Intrapleural pressure of an individual is
B. Cardiac tamponade                             being measured. In what phase did he hold
C. Emergency stage of hyperfunction and          his breath if the pressure is - 25 cm H2O?
hypertrophy                                      A.-
D. Cardiosclerosis                               E. Forced expiration
E. Tonogenic dilatation                          C. Forced inspiration
71. A physician examined a patient and           D. Quiet expiration
found inguinal hernia. Through what              E. Quiet inspiration
anatomic formation does it penetrate into        77. A 37-year-old man was admitted
the skin?                                        to the surgical department with the
A. Lacuna musculorum                             symptoms of acute pancreatitis: vomiting,
B. Anulus inguinalis super ficialis              diarrhea, bradycardia, hypotention.
C. Canalis adductorius                           weakness, dehydration of the organism.
D. Anulus femoralis                              What medicine should be used first of all?
E. Hiatus saphenus                               A. No-spa
72. A 60-year-old man felt asleep after          B. Etaperazine
cerebral hemorrhage for a long time.             C. Ephedrine
Damage of what structure caused this state?      D. Platyphylline
A. Cortex of the large hemispheres               E. Contrycal
B. Hippocampus                                   78. The alternate usage or dichlotiazide,
C. Nuclears of the cerebral nerves               etacrin acid and lasex did not influence
D. Reticular formation                           diuretically upon the patient with marked
E. Black substance                               peripheral edemata. The aldosterone rate in
73. A 25-year-old woman with red and itchy       the blood is increased. Indicate which medicine
eczematoid dermatitis visits your office. She    should be prescribed:
had a dental procedure one day earlier with      A. Urea
administration of a local anesthetic. There      B. Amilorid
were no other findings, although she indicated   C. Clopamid
that she had a history of allergic reactions.    D. Spironolacton
Which of the following drugs is most likely      E. Mannit
involved?                                        79. A person has steady HR not exceeding
A. Lidocaine                                     40 bpm. What is the pacemaker of this
B. Cocaine                                       person's heart rhythm?
C. Bupivacaine                                   A. Branches of His' bundle
D. Procaine                                      B. Purkinje's fibers
E. Etidocaine                                    C. Atrioventricular node
74. A patient with suspicion on epidemic         D. Sinoatrial node
typhus was admitted to the hospital. Some        E. His' bundle
arachnids and insects have been found in his     80. A 34-year-old patient was diagnosed with
flat. Which , of them, may be a carrier of the   chronic glomerulonephritis 3 years ago.
pathogen or epidemic typhus?                     Edemata have developed within the last 6
A. Spiders                                       month’s. What caused the edemata?
B. Cockroaches                                   A. Hyperosmolarity of plasma
C. Bed-bugs                                      B. Hyperproduction of vasopressin
D. Lice                                          C. Proteinuria
E. Houseflies                                    D. Liver dysfunction of protein formation
75. A person has reduced diuresis,               E. Hyperaldosteronism
hypernatremia, hypokalemia. Hypersecretion       81. A 5-year-old child who often fells ill
 of what hormone can cause such changes?         with respiratory diseases has eczematous
appearances after consumption of some               hyperemia and skin edema in few hours
food products, tendency to prolonged                after burn. What mechanism strengthens
course of inflammatory processes. What kind         destructive effect in the inflammation area?
of diathesis can be suspected in this case?         A. Diapedesis of erythrocytes
A. Asthenic                                         B. Proliferation of fibroblasts
B. Lymphohypoplastic                                C. Primary alteration
C. Arthritism                                       D. Emigration of lymphocytes
D. Exudative-catharral                              E. Secondary alteration
E. Hemmorhagic                                      87. A patient's blood was analyzed and
82. An 18-year-old patient has enlarged             the decreased erythrocyte's sedimentation
inguinal lymph nodes, they are painless,            rate (ESR) was discovered. What disease
thickened on palpation. In the area of genital      from the listed below is accompanied with
mucous membrane there is a small sized              decreased ESR?
ulcer with thickened edges and                      A. Polycytemia
“laquer” bottom of greyish color. What is           B. Myocardial infarction
the most probable diagnosis?                        C. Vitamin B deficiency
A. Syphilis                                         D. Hepatitis
B. Trophic ulcer                                    E. Splenomegaly
C. Tuberculosis                                     88. A considerable increase of activity of
D. Gonorrhea                                        MB-forms of CPK (creatinephosphokinase)
E. Lepra                                            and LDH-1 was revealed on the examination
83. A 56-year-old patient complaining of thirst     of patient's blood. What is the most likely
and frequent urination was diagnosed with           pathology?
diabete mellitus. Butamin was prescribed.           A. Pancreatitis
How does the medicine act?                          B. Rheumatism
A. It relieves transport of glucose through         C. Hepatitis
the cells' membranes                                D. Myocardial infarction
B. It stimulates β-cells of Langergans' islets      E. Cholecystitis
C. It helps to absorb the glucose by the cells      89. A patient had been ill with bronchial
of the organism tissues                             asthma for many years and died from
D. It inhibits α-cells of Langergans" islets        asthmatic fit. Histologic lung examination
E. It inhibits absorption of glucose in the         revealed: lumen of bronchioles and small
intestines                                          bronches contain a lot of mucus with some
84. A l-year-old child with symptoms of             eosinophils, there is sclerosis of alveolar
muscle involvement was admitted to the              septums, dilatation of alveole lumen. What
hospital. Examination revealed carnitine            mechanism of development of hypersensibility
deficiency in his muscles. What process             reaction took place?
disturbance is the biochemical basis of this        A. Cytolysis determined by lymphocytes
pathology?                                          B. Cytotoxic reaction
A. Actin and myosin synthesis                       C. Granulomatosis
B. Regulation of Ca2+ level in mitochondrion’s      D. Immunocomplex reaction
C. Substrate phosphorylation                        E. Reagin reaction -
D. Lactic acid utilization                          90. The process of heart transplantation
E. Transporting of fatty acids to mitochondrion’s   determined the viability of myocardlial cells.
85. A patient with tissue trauma was taken          The determination of what myocardium
a blood sample for the determination of             parameter is the most important?
blood clotting parameters. Specify the right        A. Rest potential of cardiomyocytes
sequence of extrinsic pathway activation:           B. Heart temperature
A. IV - VIIa - Xa                                   C. Concentration of Ca-ions in heart vessels
B. IV - VIII: TF - Xa                               D. Concentration of oxygen in heart vessels
C. III - VIII: TF - Xa                              E. Concentration of calcium-ions in myofibrils
D. III - IV - Xa                                    91. Part of the DNA chain turned 180 degree
E. III - VIIa - Xa                                  as a result of gamma radiation. What type of
86. Necrosis focus appeared in the area of          mutation took place in the DNA chain?
A. Doubling                                      C. Autosomal dominant
B. Deletion                                      D. X-linked dominant
C. Translocation                                 E. X-linked recessive
D. Replication                                   97. A 45-year-old woman suffers from
E. Inversion                                     seasonal allergic rhinitis caused by the
92. An individual is characterized by            ambrosia blossoming. What medicine from
rounded face, broad forehead, a mongolian        the stabilizer of the adipose cells group can
type of eyelid fold, flattened nasal c bridge,   be used for prevention of this disease?
permanently open mouth, projecting               A. Diazoline
lower lip, protruding tongue, short neck, flat   B. Tavegyl
hands, and stubby fingers. What diagnosis        C. Phencarol
can be put to the patient?                       D. Dimedrol
A. Klinefelter's syndrome                        E. Ketotifen
B. Supermales                                    98. After a serious viral infection a 3-year-old
C. Alkaptonuria                                  child has, repeated vomiting, loss
D. Down’s syndrome                               of consciousness, convulsions. Examination
E. Turner's syndrome                             revealed hyperammoniemia. What may
93. Autopsy of a man who died from               have caused changes of biochemical blood
chronic cardiovascular collapse revealed         indices of this child?
"tiger heart". Sidewards of endocardium          A. Inhibited activity of transamination
a yellowish-white banding can be seen;           enzymes
myocardium is dull, dark-yellow. What            B. Disorder of ammonia neutralization in
process caused this pathology?                   ornithinic cycle
A. Carbohydrate degeneration                     C. Disorder of biogenic amines neutralization
B. Fatty parenchymatous degeneration             D. Activated processes of aminoacids
C. Amyloidosis                                   decarboxylation
D. Hyaline degeneration                          E. Increased putrefaction of proteins in
E. Fatty vascular-stromal degeneratioin          intestines
94. Children often have heavy nasal breathing    99. Part of alveoles of a preterm infant didn't
resulting from excessive development             spread because of enhanced elastic recoil of
of lymphoid tissue of pharyngeal mucous          lungs. How can this recoil be reduced?
membrane. What tonsils growth may cause          A. By fluid suction from the respiratory tracts
this effect?                                     B. By glycose introduction
A. All above mentioned tonsils                   C. By surfactant introduction
B. Tonsilla palatina                             D. By artificial pulmonary ventilation
C. Tonsilla pharyngea                            E. By pure oxygene inhalation
D. Tonsilla tubaria                              100. A mother of a newborn complains
E. Tonsilla lingualis                            of her baby's constant belching with undigested
95. ATP synthesis is totaly blocked in a         milk. Which developmental anomaly
cell. How will the value of membrane rest        is it an evidence of?
potential change?                                A. Faux lupinum
A. It will disappear                             B. Esophageal atresia
B. First it will decrease, then increase         C. Labium leporiurn
C. First it will increase, then decrease         D. Anal atresia
D. It will be considerably increased             E. Esophageal fistula
E. It will be slightly increased                 101. During the operation on the hip joint of
96. A genetics specialist analyzed the           a 5-year-old child her ligament was damaged
genealogy of a family and found that both        which caused bleeding. What ligament was
males and females may have the illness, not      damaged?
across all the generations, and that healthy     A. Perpendicular of the acetabule
parents may have ill children. What is the       B. Iliofemoral
type of illness inheritance?                     C. Pubofemoral
A. Y-linked                                      D. Ischiofemoral
B. Autosomal recessive                           E. The head of the thigh
102. When a patient with traumatic impairment         E. Rifampicin
of the brain was examined, it was discovered          107. To anaesthetize the surgical treatment
that he had stopped to distinguish                    of burn surface, a patient was intravenously
displacement of an object on the skin. What           injected a medication for short-acting
part of the brain was damaged?                        narcosis. 1 minute later the patient being
A. Frontal zone                                       under anaesthesia showed increased blood
B. Occipital zone of the cortex                       pressure, tachycardia, increased tone of
C. Frontal central gurus                              skeletal muscles: reflexes remained. After
D. Posterior central gurus                            recovering from anaesthesia the patient
E. Parietal zone of the cortex                        had disorientation and visual hallucinations.
103. While having the dinner the child                What medication was the patient injected?
choked and aspirated the food. Meavy                  A. Ketamine
cough has started, skin and mucose are                B. Nitrous oxide
cyanotic, pulse is rapid, respiration is              C. Diethyl ether
infrequent, expiration is prolonged. What             D. Sombrevin
disorder of the external respiration has the child?   E. Thiopental sodium
A Stenotic respiration                                108. Healthy parents have got a fair-haired,
B. Stage of inspiratory dyspnea on asphyxia           blue-eyed girl. Irritability, anxiety, sleep and
C. Stage of expiratory dyspnea on asphyxia            feeding disturbance developed in the first
D. Biot's respiration                                 months of the infant's life. Neurological
E. Alternating respiration                            examination revealed developmental
104. The ovary specimen colored with                  lag. What method of genetic investigation
hematoxylin-eosin contains a follicle, in which       should be used for the exact diagnosis?
cubic-shaped follicle epithelium cells are            A. Gemellary
placed in 1-2 layers, and scarlet membrane is         B. Biochemical
seen around the ovocyte. Name this follicle:          C. Population-statistical
A. Atretic                                            D. Genealogical
B. Primary                                            E. Cytological
C. Secondary                                          109. A large-scale reaction with
D. Mature                                             parapertussis and pertussis diagnosticums
E. Primordial                                         was made in order to make serological
105. A 58-year-old patient with acute cardiac         diagnostics of the whooping cough. At the
insufficiency has decreased volume of daily           bottom of the test-tubes with diagnosticum
urine - oliguria. What is the mechanism of            of Bordetella parapertussis a granular
this phenomenon?                                      sediment formed. What antibodies did this
A. Decreased number of functioning                    reaction reveal?
glomerules                                            A. Antitoxins
B. Drop of oncotic blood pressure                     B. Precipitins
C. Rise of hydrostatic blood pressure 111             C. Opsonins
capillars                                             D. Agglutinins
D. Reduced permeamility of renal filter               E. Bacteriolysins
E. Decreased glomerular filtration                    110. A patient, who suffers from congenital
106. A 35-year-old man under the treatment            erythropoietic porphyria, has skin
for pulmonary tuberculosis has acute pain             photosensitivity. The accumulation of what
onset of right big toe, swelling and lowgrade         compound in the skin cells can cause it?
fever. The gouty arthritis was diagnosed              A. Uroporphyrinogen 1
and high serum uric acid level                        B. Heme
was found. Which of the following                     C. Uroporphyrinogen 2
antituberculous drugs are known for                   D. Protoporphyrin
 causing high uric acid levels?                       E. Coproporphyrinogen 3
A. Pyrazinamide                                       111. As a result of the damage of one of
B. Thiacetazone                                       the Atomic Power Plant reactors the runout
C. Aminosalicylic acid                                of radioelements took place. People
D. Cycloserine                                        in the high-radiation area were radiated
with approximately 250-300 r. They were             have been blocked on an isolated rabbit’s
immediately hospitalized. What changes in           heart. What changes in the heart's activity
the blood count would be typical for the victims?   can result from it?
A. Anemia                                           A. Heart stops in diastole
B. Lymphopenia                                      B. Heart stops in systole
C. Thrombopenia                                     C. Decreased hear beat rate
D. Neutropenia                                      D. Decreased rate and force of heart beat
E. Leukopenia                                       E. Decreased force of the contraction
112. The pulmonalis embolism has suddenly           118. Objective examination of a patient
developed in a 40-year-old patient with             revealed: slender figure, big skull, highly
opened fracture of the hip. Choose the possible     developed frontal region of face, short
kind of embolism:                                   extremities. What constitutional type is it
A. Fat                                              characteristic for?
B. Thrombus-embolus                                 A. Muscular
C. Air                                              B. Respiratory
D. Foreign body                                     C. Cerebral
E. Tissue                                           D. Digestive
113. Nowadays about 50 minor bases have             E. Mixed
been found in the t-RNA structure besides           119. Blood sampling for the haematology
the main four nitrogenous bases. Choose the         is recommended to carry out on an empty
minor nitrogenous base:                             stomach and in the morning. What changes
A. Cysteine                                         in blood formula are possible if blood
B. Dihydrouracil                                    sampling was carried out after food intake?
C. Uracil                                           A. Increase of erythrocyte number
D. Cytosine                                         B. Increase of plasm proteins
E. Adenine                                          C. Increase of leukocyte number
114. A patient has elbow joint trauma with          D. Decrease of thrombocyte number
avulsion of medial epicondyle of humerus.           E. Decrease of erythrocyte number
What nerve can be damaged in this trauma?           120. 40-year-old patient complains of
A. Medial cutaneous nerve of forearm                intensive heartbeats, sweating, nausea, visual
B. Cardiac cutaneous nerve                          impairment, arm tremor, hypertension.
C. Musculocutaneous nerve                           From his anamnesis: 2 years ago he
D. Radial                                           was diagnosed with pheochromocytoma.
E. Ulnar                                            Hyperproduction of what hormones causes
115. Diabetes mellitus causes ketosis as a          the given pathology?
result of activated oxidation of fatty acids.       A. Catecholamines
What disorders of acid-base equilibrium             B. Aldosterone
may be caused by excessive accumulation of          C. Glucocorticoids
ketone bodies in blood?                             D. Thyroidal hormones
A. Metabolic alkalosis                              E. ACTH
B. Metabolic acidosis                               121. Microspecimen of spinal cord contains a
C. Any changes woun't happen                        nucleus that should be analyzed. lts
D. Respiratory acidosis                             neurons form motor endings in the skeletal
E. Respiratory alkalosis                            muscles. What nucleus of spinal cord is
116. A 30-vear-old woman was diagnosed              meant?
with insufficiency of exocrinous function of        A. Proper nucleus of the posterior horn
pancreas. Hydrolisis of what nutrients will         B. Intermediate lateral nucleus
be disturbed?                                       C. Thoracic nucleus
A. Proteins                                         D. Proper nucleus of gray substance
B. Fats, carbohydrates                              E. Proper nucleus of the anterior horn
C. Proteins, carbohydrates                          122. Parents with an ill child consulted
D. Proteins. fats                                   an infectionist. They had been working in
E. Proteins, fats, carbohydrates                    one of Asian countries for a long time.
117. The calcium canals of cardiomyocytes           The child has sallow skin, loss of appetite,
laxity, enlarged liver, spleen, peripheral       B. Uric acid
lymph nodes. What protozoal illness can be       C. Glucose
suspected?                                       D. Common lipids
A. Balantidiasis                                 E. Mineral salts
B. Lambliasis                                    128. A 50-year-old patient was injured on
C. Toxoplasmosis                                 the occipital region of the head. The dosed
D. Visceral leishmaniasis                        skull trauma was diagnosed. She was taken
E. Amebiasis                                     to the hospital. The medical examination:
123. In the specimen of one of the parts         deregulation of walking and balance.
of respiratory system a tubular organ was        trembling of arms. What part of brain was
found. It has low epithelium, well developed     injured?
muscular tunic, glands and cartilage are         A. The spinal cord
absent. Name this organ:                         B. The mind-brain
A. Minor bronchs                                 C. The inter-brain
B. Major bronchs                                 D. The cerebellum
C. Median bronchs                                E. The medulla oblongata
D. Trachea                                       129. A woman with 0 (l) blood group
E. Larynx                                        has born a child with AB blood group.
124. A 63-year-old woman shows symptoms          This woman's husband has A blood group.
of rheumaioid arthritis. The increase of what    What genetic interaction explains this
blood indices could bc the most significant      phenomenon?
for proving the diagnosis?                       A. Polymery
A. Additive glycosaminoglycans                   B. Recessive epistasis
B. Lipoproteids                                  C. Codominance
C. Acid phosphatase                              D. Incomplete dominance
D. R-glycosidase                                 E. Complementation
E. General cholesterol                           130. Patients with bile ducts obstruction
125. A person felt thirsty after staying under   suffer from inhibition of blood coagulation,
the conditions of hot weather for a long time.   bleedings as a result of low level of vitamin
Signals of what receptors caused it first        assimilation. What vitamin is in deficiency?
of all?                                          A. E
A. Osmoreceptors of liver                        B. D
B. Glucoreceptors of hypothalamus                C. K
C. Osmoreceptors of hypothalamus                 D. A
D. Baroreceptors of aortic arch                  E. Carotene
E. Sodium receptors of hypothalamus              131. A businessman came to India from
126. A 57-year-old patient was admitted          South America. On examination the physician
to the gastroenterological department with       found that the patient was suffering
suspicion of Zollinger- Ellison syndrom          from sleeping-sickness.What was the way of
because of rapid increase of gastrin level       invasion?
in the blood scrum. What is the most             A. As a result of mosquito`s bites
probable disorder of the secretory function      B. Through dirty hands
of stomach?                                      C. With contaminated fruits and vegetables
A. Hyperacidic hyposecretion                     D. After contact with sick dogs
B. Hypoacidic hyposecretion                      E. As a result of bug's bites
C. Hyperacidic hypersecretion                    132. Examination of a newborn boy's genitals
D. Hypoacidic hypersecretion                     revealed a cleft of urethra that opens
E. Achylia                                       on the inferior surface of his penis. What
127. A patient with serious damage of            developmental anomaly is meant?
muscular tissue was admitted to the              A. Monorchism
traumatological department. What biochemical     B. Hypospadia
urine index will be increased in                 C. Epispadia
this case?                                       D. Cryptorchism
A. Creatinine                                    E. Hermaphroditism
133. A pregnant woman had been having              performed to reveal latent toxoplasmosis
toxicosis with severe repeated vomiting            and chronic gonorrhoea of the patient?
for 24 hours. In the end of the day there          A.. IFA - Immunofluorescence assay
appeared tetanic convulsions and fluid loss.       B. (R)CFT- Reiter's complement fixation
What shift of acid-base state caused these         test
changes?                                           C. RTHA - Reverse indirect hemagglutination
A. Excretory alkalosis                             assay
B. Excretory acidosis                              D. Immunoblot analysis
C. Metabolic acidosis                              E. RDHA - Reverse direct hemagglutination
D. Gaseous alkalosis                               assay
E. Gaseous acidosis                                139. A 16-year-old boy was performed an
134. Live vaccine was injected into the            appendectomy. He has been hospitalized
human body. Increasing activity of what            for right lower quadrant abdominal pain
connective tissue cells can be expected?           within 18 hours. The surgical specimen is
A. Adipocytes and adventitious cells               edematous and erythematous. Infiltration
B. Macrophages and fibroblasts                     by what of the following ceils is the most
C. Fibroblasts and labrocytes                      typical for the process occurring here?
D. Pigmentocytes and pericytes                     A. Monocytes
E. Plasmocytes and lymphocytes                     B. Limphocytes
135. On autopsy of the 58-year-old man it          C. Basophils
was revealed: mitral valve is deformed,            D. Eosinophils
thickened, not totally closed. Microscopically:    E. Neutrophils
foci of collagen fibers are eosinophilic, have     140. A patient who has been treated
positive fibrin reaction. The most likely it is:   with diazepam on account of neurosis
A. Fibrinoid swelling                              complains of toothache. Doctor administered
B. Amyloidosis                                     him an analgetic, but its dose was
C. Mucoid swelling                                 lower then average therapeutic dose. What
D. Hyalinosis                                      phenomenon did the doctor take into
E. Fibrinoid inflammation                          account while prescribing the patient an
136. A 38-year-old woman was admitted to           underdose?
the admission-diagnostic department with           A. Drug dependence
uterine bleeding. What are the most likely         B. Cumulation
changes of blood?                                  C. Tolerance
A. Leukopenia                                      D. Summation
B. Leucocytosis                                    E. Potentation
C. Reduction of haematocrite rate                  141. Glomerular filtration rate (GFR)
D. Increase of haematocrite rate                   increased by 20% as a result of prolonged
E. Polycythemia                                    starvation of an individual. The most evident
137. A 60-year-old patient was admitted to         cause of filtration changes under this
the surgical department because of infection       conditions is:
caused by blue pus bacillus (Pseudomonas           A. Increase of filtration coefficient
aeruginosa) which is sensitive to penicillin       B. Increase of systemic blood pressure
antibiotics. Indicate which of the given           C. Decrease of oncotic pressure of blood
penicillin’s has marked activity to the            plasma
Pseudomonas aeruginosa?                            D. Increase of penetration of the renal filter
A. Methicillin                                     E. Increase of renal plasma stream
B. Phenoxymethylpenicillin                         142. The preventive radio protector was given
C. Carbenicillin disodium                          to worker of a nuclear power station.
D. Oxacillin                                       What mechanism from the below mentioned
E. Benzylpeniciilin                                is considered to be the male mechanism of
138. A patient who came to the doctor              radioprotection?
because of his infertility was administered        A. Prevention of tissue's hypoxia
to make tests for toxoplasmosis and chronic        B. Inhibition of free radicals formation
gonorrhoea. Which reaction should be               C. Activation of oxidation reactions
D. Increasing of tissue blood supply               bar. Objectively: pain while moving upper
E. Increasing of respiration                       extremity, reduced pronation and adduction
143. Examination of initial molecular              functions. Sprain of what muscle can be
structure revealed substitution of the             observed here?
glutamic acid by valine. What inherited            A. M. latissimus dorsi
pathology is it typical for?                       B. M. subscapularis
A. Hemoglobinosis                                  C. M. romboideus major
B. Thalassemia                                     D. M. levator scapulae
C. Minkowsky-Shauffard disease                     E. M. trapezius
D. Favism                                          149. A young man has a painlessl formation
E. Sickle-cell anemia                              without marked borders in the soft tissues of
144. A patient experienced a sudden                his thigh. On the tissue bioptate the formation
temperature rise up to 39°C. After 6 hours         looks like flesh of fish and consists of
the temperature normalized. On the 2-nd            immature fibroblast-like cells with multiple
day the attack recurred: in the period of          mitosis growing through the muscles. What
paroxysm the temperature reached 41°C,             is the most likely diagnosis?
apyrexial period began after 8 hours. What         A. Fibrosarcoma
type of temperature profile is it?                 B. Fibroma
A. Recurrent                                       C. Cancer
B. Septic                                          D. Myosarcoma
C. Continued                                       E. Myoma
D. lntermitting                                    150. Analysis of amniotic fluid that was
E. Hectic                                          obtained as a result of amniocentesis
145. Albinos can't stand sun impact - they         (puncture of amniotic sac) revealed cells
don't aquire sun-tan but get sunburns. Disturbed   the nuclei of which contain sex chromatin
metabolism of what aminoacid                       (Barr's body). What can it be evidence of?
underlies this phenomenon?                         A, Polyploidy
A. Glutamic acid                                   B. Trisomy
B. Histidine                                       C. Genetic disorders of fetus development
C. Tryptophan                                      D. Development of female fetus
D. Phenilalanine                                   E. Development of male fetus
E. Methionine                                      151. A 50-year-old patient with typhoid
146. A child has got galactosemia.                 fever was treated with Levomycetin,
Concentration of glucose, in blood has not         next day his condition became worse,
considerably changed. Deficiency of what           temperature rose to 39,6°C. What caused
enzyme caused this illness?                        the complication?
A. Galactose-l-phosphate uridyltransferase         A. The effect of endotoxin agent
B. Hexokinase                                      B. Reinfection
C. Galactokinase                                   C. Secondary infection addition
D. Amylo-1.6-g1ucosidase                           D. Irresponsiveness of an agent to the
E. Phosphoglucomutase                              levomycetin
147. A. patient with suspected diphtheria          E. Allergic reaction
went through bacterioscopic examination.           152. A patient with infectious mononucleosis
Examination of throat swab revealed rodshaped      had been taking glucocorticoids for two
bacteria with volutin granules.                    weeks. He was brought into remission, but
What etiotropic preparation should                 he fell ill with acute attack of chronic tonsillitis.
 be chosen in this case?                           What action of glucocorticoids caused
A. Bacteriophage                                   this complication?
B. Interferon                                      A. Antiallergic
C. Antidiphtheric antitoxic scrum                  B. Immunosuppressive
D. Diphtheria antitoxin                            C. Anti-inflammatory
E. Eubiotic                                        D. Antishock
148. A young man felt acute pain in the back       E. Antitoxic
during active drawing up on the horizontal         153. A 40-year-old woman was admitted to
the infectious diseases department with high         involvement of intestines, liver, lungs. What
body temperature. Objectively: marked                drug should be prescribed?
meningeal symptoms. A spinal cord punction           A. Enteroseptol
was made. What anatomic formation was                B. Tetracycline
puncturated?                                         C. Quiniofone
A. Carum trigeminale                                 D. Quingamine
B. Spatium subarachnoideum                           E. Metronidasol
C. Cisterna cerebellomedularis posterior             159. Tuberculosis can be treated by means
D. Spatium subdurale                                 of combined chemotherapy that includes
E. Spatium epidurale                                 substances with different mechanisms of
154. A patient has got a spasm of smooth             action. What antituberculous medication
muscles of bronchi. Activators of what               inhibits transcription of RNA into DNA in
membrane cytoreceptors are phisiologically           mycobacteria?
reasoned to stop an attack?                          A. Isoniazid
A. β-adrenoreceptors                                 B. Para-aminosalicylic acid
B. M-cholinoreccptors                                C. Rifampicin
C. α- and β-adrenoreceptors                          D. Ethionamide
D. H-cholinoreceptors                                E. Streptomycin
E. α -adrenoreceptors                                160. A patient who was previously ill with
155. A patient had been taking glucocorticoids for   mastectomy as a result of breast cancer was
a long time. When the preparation                    prescribed radiation therapy. What vitamin
was withdrawn he developed the symptoms              preparation has marked radioprotective
of disease aggravation, decreased blood              action caused by antioxidant activity?
pressure and weakness. What is the reason            A. Ergocalciferol
of this condition?                                   B. Tocopherol acetate
A. Appearance of adrenal insufficiency               C. Thiamine chloride
B. Hyperproduction of ACTH                           D. Ribof1avin
C. Cumulation                                        E. Folic acid
D. Sensibilization                                   161. Inhibition of α-motoneuron of the
E. Habituation                                       extensor muscles was noticed after stimulation
156. The energy inputs of a healthy man              of α-motoneuron of the flexor
have been measured. In what state was the            muscles during the experiment on the spinal
patient if his energy inputs were less than          cord. What type of inhibition is this process
the main exchange?                                   based upon?
A. Rest                                              A. Lateral
B. Nervous exertion                                  B. Recurrent
C. Easy work                                         C. Thiamine chloride
D. Sleep                                             D. Presynaptic
E. Calmness                                          E. Reciprocal
157. An 18-year-old patient came to the              162. While shifting the gaze to the closely
outpatient department with the complaints of         situated object the refracting power of eye's
bleeding trauma in the vestibule of his nose.        optical mediums will increase by 10 diopters.
On examination: the mechanical injury of             It results from changing of such eye
the mucous layer of the vestibule without            structure:
continuation into nasal cavity proper. What          A. Vitreous body
is the boundary between the vestibule and            B. Liquid of the anterior chamber of eye
nasal cavity proper?                                 C Cornea
A. Nasal limen                                       D. Lens
B. Nasal roller                                      E. Muscle that dilatates pupil
C. Nostrils                                          163. In course of metabolic process active
D. Nasal septa                                       forms of oxygen including superoxide anion
E. Choanes                                           radical are formed in the human body. By
158. A 52-year-old patient has the following         means of what enzyme is this anion inactivated?
diagnosis: systemic amebiasis with                   A. Peroxidase
B. Superoxide disffiutase                         ventricular arrhythmias was admitted to
C. Catalase                                       the hospital. The patient is taking timolol
D. Glutathioneperoxidase                          drops for glaucoma, daily insulin injections
E. Glutathionereductase                           for diabetes mellitus, and an ACE inhibitor
164. A student is writing a thorough              for hypertension. You have decided to use
summary of a lecture. Quality of summarizing      phenytoin instead of procainamide. What is
has considerably worsened when his neighbors      the reason?
started talking. What kind of inhibition          A. The local anesthetic effect of procainamide
in the cerebral cortex caused this effect?        would potentiate diabetes
A. Protective                                     B. The cholinergic effects of procainamide
B. Fading                                         would aggravate the diabetes
C. Differentiated                                 C. The hypertensive effects of procainarmide
D. Delayed                                        would aggravate the hypertension
E. External                                       D. The anticholinergic effect of procainamide
165. Analeptical remedy of ret1ective type        would aggravate glaucoma
from the H -cholinomimetics group was given       E. The local anesthetic effect of procainamide
to the patient for restoration of breathing       would aggravate the hypertension
after poisoning with carbon monoxide.             169. Tuberculine was introduced intracutaneously
What medicine was prescribed to the patient?      to the child for tuberculin test. Marked
A. Lobeline hydrochloride                         hyperemia,
B. Mesaton                                        tissue infiltration developed on
C. Adrenaline hydrochloride                       the place of injection in 24 hours. What
D. Atropine sulphate                              mechanism caused these modifications?
E. Pentamin                                       A. Immunocomplex cytotoxity
166. It is planned to use the territory of an     B. Antibody cytotoxity
old cattle burial ground (which is not used       C. Granuloma formation
for more than 50 years) for building houses.      D. Reagin type cytotoxity
But ground analysis revealed presence of          E. Cell cytotoxity
the pathogen of a very dangerous illness.         170. Concentration of pyruvate is increased
Which of the indicated microorganisms is likely   in the patient's blood, the most of which is
to remain in the ground for such a long           excreted with urine. What avitaminosis has
time?                                             the patient?
A. Brucella abortus                               A. Avitaminosis B6
B. Mycobacteriurn bovis                           B. Avitaminosis E
C. Francisella tubarensis                         C. Avitaminosis B2
D. Bacillus anthracis                             D. Avitaminosis B1
E. Yersinia pestis                                E. Avitaminosis B3
167. A 22-year-old patient was admitted           171. Upper neck node or Sympathetic trunk
to the hospital with Complaints of heavy          was removed from the rabbit on experiment.
nasal breathing. During the examination           Reddening and increased temperature of
of her nasal cavity the doctors found thickened   the skin of head is observed. What disorder
mucous membrane, a lot of mucus                   of peripheral circulation of the blood has
and nodular infiltrates without erosions in       developed?
the nose. The nasal rhinosc1eroma was             A. Neurotonic arterial hyperemia
diagnosed.                                        B. Neuroparalytic arterial hyperemia
The biopsy was taken. What typical                C. Stasis
morphological                                     D. Venous hyperemia
changes may be found?                             E. Metabolic arterial hyperemia
A. Granulomas with Larnghan's cells               172. Diagnostic scraping was performed
B. Granulomas with foreign bady cells             to the woman with dysfunctional uterine
C. Interstitial inflammation                      bleeding. Mu1tip1e convoluted glands,
D. Granulomas with Virchow`s cells                ganglially dilated lumens of some glands
E. Granulomas with Mikuliez's cel1s               were revealed histologically in the scrape.
168 A 55-year-old patient with continuing         Name the type or general pathological
process in endometry:                                  notes that he needed to buy bigger shoes
A. Hypertrophic excrescence                            three times. What is the main reason of such
B. Metaplasia                                          disproportional enlargement of different
C. Displasia                                           parts of the body?
D. Hyperplasia glandulocystica                         A. Joints dystrophy development
E. Atrophy                                             B. Increased sensitivity of the tissues to
173. While enrolling a child to school                 growth hormone
Mantu`s test was made to define whether                C. Joints chronic inflammation development
revaccination was needed. The test result is           D. Increased sensitivity of the tissues to
negative. What does this test result mean?             insulin
A. Absence of antitoxic immunity to the                E. Cartilaginous tissue proliferation under
tuberculosis                                           growth hormone influence
B. Presence of cell immunity to the                    178. A 54-vear-old man was admitted to the
tuberculosis                                           hospital with complaints of pain in the right
C. Absence of cell immunity to the                     subcostal region, vomiting with blood.
tuberculosis                                           Objectively: enlarged liver, varicose veins in
D. Presence of antibodies for tubercle bacillus        the stomach and esophagus. Disfunction of
E. Absence of antibodies for tubercle bacillus         what vessel is likely to have taken place?
174. Buffer capacity of a worker's blood was           A. Vena cava superior
decreased due to exhausting muscular work.             B. Vena porta
By coming of what acid substance in the                C. Vena hepatica
blood can this state be explained?                     D. /ena cava inferior
A. α-ketoglutarate                                     E. Aorta abdominalis
B. Lactate                                             179. Under SOme diseases it iS observed
C. 1,3-bisphosphoglycerate                             aldosteronism accompanied by hypertension
D.3-phosphoglycerate                                   and edema due to sodium retention in
E. Pyruvate                                            the organism. What organ of the internal
175. A man who was bitten by the unknown               secretion is affected under aldosteronism?
dog applied to the surgeon. Wide ragged                A. Testicle
woundes were localized on the face. What               B. Pancreas
curative-prophylactic aid should be given to           C. Ovaries
prevent rabies?                                        D. Adrenal glands
A. Prescribe combined antibiotic therapy               E. Hypophysis
B. Hospitalize the patient and keep under              180. Microscopic examination of the sputum
the doctor`s                                           of a patient with pneumonia occasionally
C. Start immunisation with rabies vaccine              revealed some larvae. Eosinophiles were
D. Immediately inject normal gammaglobulin             detected on blood examination. What
E. Immediate injection of DPT(Diphtheria,              helminthiasis can be diagnosed?
Pertusis, Tetanus) vaccine                             A. Enterobiosis
176. A 25-year-old patient complained of               B. Opisthorchosis
the decreased vision. Accommodation disorders,         C. Ascariasis
dilated pupil, lack of reaction for                    D. Trichocephaliasis
the light were revealed on examination.                E. Paragonimiasis
What muscles function is disturbed?                    181. A duodenal content smear of a patient
A. Pupil dilating muscle, ciliary                      with indigestion contains protosoa 10-18
B. Pupil narrowing muscle, ciliary                     mcm large. They have piriform bodies, 4 pairs
C. Pupil narrowing and dilating muscles                of filaments, two symmetrically located
D. Inferior oblique muscle, ciliary                    nuclei in the broadened part of body. What
E. Lateral rectus muscle, pupil narrowing              kind of the lowest organisms is it?
177. A 46-year-old patient has complained              A. Larnblia
of headache, fatigue, thirst, pains in the spine and   B. Dysentery ameba
joints for the last 2 years. Clinically                C. Trichomonas
observed disproportional enlargement of                D. Intestinal ameba
hands: feet, nose, superciliary arches. He             E. Balantidium
182. RNA-polymerase B(II) is blocked due               A. Glutamic
to amanitine poisoning (poison of deathcup).           B. Tryptophan
It disturbs:                                           C. Tyrosine
A. Synthesis of t-RNA                                  D. Valine
B. Synthesis of m-RNA                                  E. Lisine
C. Maturation of m-RNA                                 188. Microscopic examination of the
D. Primers synthesis                                   enlarged neck gland of a 14-year-old girl
E. Reverse transcription                               revealed destruction of the tissue structure
183. As a result of craniocerebral trauma              of the node, absence of the lymph folli-
a patient reveals the following symptoms:              cles, sclerotic areas and necrosis foci, cell
intention tremor, dysmetry, adiadochokinesis,          constitution of the node is polymorphous,
dysarthria. What structure of the brain                lymphocites. eosinophiles, big atypical
is injured?                                            cells with multilobular nuclei (Beresovsky-
A. Black substance                                     Sternberg cells) and mononuclear cells of
B. Motor cortex                                        the large size are present. What is the most
C. Striatum                                            likely diagnosis?
D. Pale'sphere                                         A. Chronic lympholeucosis B.
E. Cerebellum                                          Lymphogranulomatosis
184. Moving of the daughter chromatids to              C. Berkitt's lymphoma
the poles of the cell is observed in the mitotically   D. Acute lympholeucosis
dividing cell. At what stage of the                    E. Fungoid mycosis
mitotic cycle is this cell?                            189. An isolated muscle of a frog is rhythmi-
A.       Interphase                                    cally irritated with electric impulses. Every
B.       Prophase                                      next impulse is in a period of relaxation from
C.       Telophase                                     the previus contraction. What contraction of the
D.       Metaphase                                     muscle occurs?
Ё. Anaphase                                            A. Single
185. Autopsy of a 12-year-old girl revealed:           B. Continuous (smooth) tetanus
multiple cutaneous hemorrhages (mostly                 C. Asynchronous
into the skin of buttocks, lower extremi-              D. Tonic
ties), serous and mucous membrane                      E. Waved tetanus
hemorrhages, cerebral hemorrhages.                     190. On autopsy it was revealed: pia mater
Adrenal glands show focal necrosis and                 of the upper parts of cerebral hemisphere
massive hemorrhages; kidneys show                      is plethoric, of yellowish-green color, soaked
necrotic nephrosis, suppurative arthritis,             with purulent and fibrose exudate, looks like
iridocyclitis, vasculitis. What is the most            a cap. What disease is it typical for?
probable diagnosis'?                                   A. Grippal meningitis
A. Epidemic typhus                                     B. Tuberculous meningitis
B. Systemic lupus erythematosus                        С Meningitis connected with typhus
С. Radiation sickness                                  D. Meningococcal meningitis
D. Periarteritis nodosa                                E. Meningitis connected with anthrax
E. Meningococcemia                                     191. A 50-year-old man has felt vague
186. A patient working at a chemical                   abdominal discomfort within past 4
plant was admitted to the toxicological                months. Physical examination revealed
department with mercury poisoning. What                no lymphadenopathy, and no abdominal
medicine should be used?                               masses or organomegaly at palpation. Bowel
A. Isonitrozin                                         sounds are heard. An abdominal CT scan
B. Naloxone                                            shows a 20 cm retroperitoneal soft tissue
C. Enterosorbent                                       mass obscuring the left psoas muscle. A
D. Unithiol                                            stool specimen tested for occult blood is
E. Activated carbon                                    negative. Which of the following neoplasm’s
187. Cerebral trauma caused increase of ammonia        is this man most likely to have?
formation. What aminoacid takes part in removal        A. Melanoma
of ammonia from cerebral tissue?                       B. Hamartoma C. Lipoma
D. Lymphoma                                        197. A woman who was infected with
E. Adenocarcinoma                                  toxoplasmosis during the pregnancy has
192. A child has inhaled a button. Where is        born a child with multiple congenital
it likely to be?                                   defects. This is a result of:
A. In the esophagus                                A. Recombination
B. In the left main bronchus                       B. Cancerogenesis
C. In the larynx                                   C. Biological mutogenesis
D. In the right main bronchus                      D. Chemical mutogenesis
 E. In the trachea                                 E. Teratogencsis
193. A 68-year-old woman can't move her            198. After a trauma a 44-year-old patient
upper and lower right extremities due to           had a rupture of left palm muscle tendons
the stroke. Muscle tone of these extremities and   and of the superficial blood vessels. After
their reflexes are increased. There are            operation and removal of the most part of
pathological reflexes. What form of paralysis      the necrotically changed muscle tissue the
is it?                                             bloodstream was normalized. What vessels
A. Monoplegia                                      have helped to restore the bloodstream?
B. Paraplegia                                      A. Aa. metacarpeae palmares
C. Tetraplegia                                     B. Aa.digitales palmares communes
D. Dissociation                                    C. Arcus palmaris superficialis
E. Hemiplegia                                      D. Aa.perforantes
194. During the fetal period of the                E. Arcus palmaris profundus
development in the vascular system of              199. A patient who has been strictly keeping
the fetus a large arterial (Botallo's)             to a certain diet for 10 days went through
duct is functioning which converts into            examination of respiratory coefficient. It
Hg.arteriosum after birth. What anatomical         was determined that it was equal 1. What
formations does this duct connect?                 have the patient been keeping to?
A. Aorta and superior vena cava                    A. With domination of proteins and fat
B. Aorta and inferior vena cava                    B. With domination of fat and carbohydrates
C. Pulmonary trunk and aorta                       C. With domination of carbohydrates
D. Right and left auricles                         D. Mixed
E. Pulmonary trunk and superior vena cava          E. With domination of proteins and
195. An intraoperational biopsy of mammal          carbohydrates
gland has revealed the signs of atypical tissue    200. A patient has tissue ischemia below the
presented by disorder of parenchyma                knee joint accompanied with intermittent
stroma proportion with domination of the           claudication. What artery occlusion should
last, gland structures of different size and       be suspected?
shape, lined with single-layered proliferative     A. Popliteal artery
epithelium. What is the most probable              B. Anterior tibial artery
diagnosis?                                         C. Proximal part of femoral artery
A. Noninfiltrative cancer                          D. Posterior tibial artery
B. Infiltrative cancer                             E. Peroneal artery
C. Papilloma
D. Fibroadenoma
E. Mastitis
196. A person was selling "homemade
pork" sausages at the market. State sanitary
inspector suspected falcification of the
sausages. What serological immune reaction
can identify food substance?
A. Agglutination test
B. Immunofluorescence test
C. Precipitation test
D. Indirect hemagglutination test
E. Complement-fixation test

More Related Content

What's hot

Krok 1 - 2015 (Physiology)
Krok 1 - 2015 (Physiology)Krok 1 - 2015 (Physiology)
Krok 1 - 2015 (Physiology)Eneutron
 
Krok 1 - 2006 Question Paper (Stomatology)
Krok 1 - 2006 Question Paper (Stomatology)Krok 1 - 2006 Question Paper (Stomatology)
Krok 1 - 2006 Question Paper (Stomatology)Eneutron
 
Krok 1 - 2015 (Microbiology)
Krok 1 - 2015 (Microbiology)Krok 1 - 2015 (Microbiology)
Krok 1 - 2015 (Microbiology)Eneutron
 
Krok 1 - 2013 Question Paper (Stomatology)
Krok 1 - 2013 Question Paper (Stomatology)Krok 1 - 2013 Question Paper (Stomatology)
Krok 1 - 2013 Question Paper (Stomatology)Eneutron
 
Krok 1 2014 - path physiology
Krok 1   2014 - path physiologyKrok 1   2014 - path physiology
Krok 1 2014 - path physiologyEneutron
 
Krok 1 - 2015 (Biology)
Krok 1 - 2015 (Biology)Krok 1 - 2015 (Biology)
Krok 1 - 2015 (Biology)Eneutron
 
Krok 1 2014 - path anatomy
Krok 1   2014 - path anatomyKrok 1   2014 - path anatomy
Krok 1 2014 - path anatomyEneutron
 
Krok 1 - 2009 Question Paper (Stomatology)
Krok 1 - 2009 Question Paper (Stomatology)Krok 1 - 2009 Question Paper (Stomatology)
Krok 1 - 2009 Question Paper (Stomatology)Eneutron
 
Krok 1 2014 - histiology
Krok 1   2014 - histiologyKrok 1   2014 - histiology
Krok 1 2014 - histiologyEneutron
 
Krok 1 - 2006 Question Paper (General medicine)
Krok 1 - 2006 Question Paper (General medicine)Krok 1 - 2006 Question Paper (General medicine)
Krok 1 - 2006 Question Paper (General medicine)Eneutron
 
Krok 1 - 2009 Question Paper (General medicine)
Krok 1 - 2009 Question Paper (General medicine)Krok 1 - 2009 Question Paper (General medicine)
Krok 1 - 2009 Question Paper (General medicine)Eneutron
 
Krok 1 - 2015 Base (General Medicine)
Krok 1 - 2015 Base (General Medicine)Krok 1 - 2015 Base (General Medicine)
Krok 1 - 2015 Base (General Medicine)E_neutron
 
Krok 1 - 2014 (Biology)
Krok 1 - 2014 (Biology)Krok 1 - 2014 (Biology)
Krok 1 - 2014 (Biology)Eneutron
 
Microbiology MCQs
Microbiology MCQsMicrobiology MCQs
Microbiology MCQsPankaj Jha
 
Krok 1 - 2011 Question Paper (Stomatology)
Krok 1 - 2011 Question Paper (Stomatology)Krok 1 - 2011 Question Paper (Stomatology)
Krok 1 - 2011 Question Paper (Stomatology)Eneutron
 
Krok 1 2014 - biology
Krok 1   2014 - biologyKrok 1   2014 - biology
Krok 1 2014 - biologyEneutron
 
Krok 1 - 2015 Question Paper (General medicine)
Krok 1 - 2015 Question Paper (General medicine)Krok 1 - 2015 Question Paper (General medicine)
Krok 1 - 2015 Question Paper (General medicine)Eneutron
 

What's hot (19)

Krok 1 - 2015 (Physiology)
Krok 1 - 2015 (Physiology)Krok 1 - 2015 (Physiology)
Krok 1 - 2015 (Physiology)
 
Krok 1 - 2006 Question Paper (Stomatology)
Krok 1 - 2006 Question Paper (Stomatology)Krok 1 - 2006 Question Paper (Stomatology)
Krok 1 - 2006 Question Paper (Stomatology)
 
Physiology
Physiology Physiology
Physiology
 
Krok 1 - 2015 (Microbiology)
Krok 1 - 2015 (Microbiology)Krok 1 - 2015 (Microbiology)
Krok 1 - 2015 (Microbiology)
 
Krok 1 - 2013 Question Paper (Stomatology)
Krok 1 - 2013 Question Paper (Stomatology)Krok 1 - 2013 Question Paper (Stomatology)
Krok 1 - 2013 Question Paper (Stomatology)
 
Krok 1 2014 - path physiology
Krok 1   2014 - path physiologyKrok 1   2014 - path physiology
Krok 1 2014 - path physiology
 
2008
20082008
2008
 
Krok 1 - 2015 (Biology)
Krok 1 - 2015 (Biology)Krok 1 - 2015 (Biology)
Krok 1 - 2015 (Biology)
 
Krok 1 2014 - path anatomy
Krok 1   2014 - path anatomyKrok 1   2014 - path anatomy
Krok 1 2014 - path anatomy
 
Krok 1 - 2009 Question Paper (Stomatology)
Krok 1 - 2009 Question Paper (Stomatology)Krok 1 - 2009 Question Paper (Stomatology)
Krok 1 - 2009 Question Paper (Stomatology)
 
Krok 1 2014 - histiology
Krok 1   2014 - histiologyKrok 1   2014 - histiology
Krok 1 2014 - histiology
 
Krok 1 - 2006 Question Paper (General medicine)
Krok 1 - 2006 Question Paper (General medicine)Krok 1 - 2006 Question Paper (General medicine)
Krok 1 - 2006 Question Paper (General medicine)
 
Krok 1 - 2009 Question Paper (General medicine)
Krok 1 - 2009 Question Paper (General medicine)Krok 1 - 2009 Question Paper (General medicine)
Krok 1 - 2009 Question Paper (General medicine)
 
Krok 1 - 2015 Base (General Medicine)
Krok 1 - 2015 Base (General Medicine)Krok 1 - 2015 Base (General Medicine)
Krok 1 - 2015 Base (General Medicine)
 
Krok 1 - 2014 (Biology)
Krok 1 - 2014 (Biology)Krok 1 - 2014 (Biology)
Krok 1 - 2014 (Biology)
 
Microbiology MCQs
Microbiology MCQsMicrobiology MCQs
Microbiology MCQs
 
Krok 1 - 2011 Question Paper (Stomatology)
Krok 1 - 2011 Question Paper (Stomatology)Krok 1 - 2011 Question Paper (Stomatology)
Krok 1 - 2011 Question Paper (Stomatology)
 
Krok 1 2014 - biology
Krok 1   2014 - biologyKrok 1   2014 - biology
Krok 1 2014 - biology
 
Krok 1 - 2015 Question Paper (General medicine)
Krok 1 - 2015 Question Paper (General medicine)Krok 1 - 2015 Question Paper (General medicine)
Krok 1 - 2015 Question Paper (General medicine)
 

Similar to 2006

Krok 1 - 2015 (Histology)
Krok 1 - 2015 (Histology)Krok 1 - 2015 (Histology)
Krok 1 - 2015 (Histology)Eneutron
 
Krok 1 - 2007 Question Paper (Stomatology)
Krok 1 - 2007 Question Paper (Stomatology)Krok 1 - 2007 Question Paper (Stomatology)
Krok 1 - 2007 Question Paper (Stomatology)Eneutron
 
Krok 1 - 2015 Question Paper (Stomatology)
Krok 1 - 2015 Question Paper (Stomatology)Krok 1 - 2015 Question Paper (Stomatology)
Krok 1 - 2015 Question Paper (Stomatology)Eneutron
 
Krok 1 - 2008 Question Paper (Stomatology)
Krok 1 - 2008 Question Paper (Stomatology)Krok 1 - 2008 Question Paper (Stomatology)
Krok 1 - 2008 Question Paper (Stomatology)Eneutron
 
Krok 1 - 2008 Question Paper (General medicine)
Krok 1 - 2008 Question Paper (General medicine)Krok 1 - 2008 Question Paper (General medicine)
Krok 1 - 2008 Question Paper (General medicine)Eneutron
 
Krok 1 Medicine - 2016 General Medicine
Krok 1 Medicine - 2016 General MedicineKrok 1 Medicine - 2016 General Medicine
Krok 1 Medicine - 2016 General MedicineEneutron
 
Krok 1 - 2011 Question Paper (General medicine)
Krok 1 - 2011 Question Paper (General medicine)Krok 1 - 2011 Question Paper (General medicine)
Krok 1 - 2011 Question Paper (General medicine)Eneutron
 
Krok 1 - 2015 (Anatomy)
Krok 1 - 2015 (Anatomy)Krok 1 - 2015 (Anatomy)
Krok 1 - 2015 (Anatomy)Eneutron
 
Krok1 stomatology - 2016
Krok1   stomatology - 2016Krok1   stomatology - 2016
Krok1 stomatology - 2016Eneutron
 
Krok 1 - 2014 Question Paper (General medicine)
Krok 1 - 2014 Question Paper (General medicine)Krok 1 - 2014 Question Paper (General medicine)
Krok 1 - 2014 Question Paper (General medicine)Eneutron
 
Krok 1 - 2015 (Path-Physiology)
Krok 1 - 2015 (Path-Physiology)Krok 1 - 2015 (Path-Physiology)
Krok 1 - 2015 (Path-Physiology)Eneutron
 
Krok 1 - 2012 Question Paper (General medicine)
Krok 1 - 2012 Question Paper (General medicine)Krok 1 - 2012 Question Paper (General medicine)
Krok 1 - 2012 Question Paper (General medicine)Eneutron
 
Krok 1 - 2010 Question Paper (General medicine)
Krok 1 - 2010 Question Paper (General medicine)Krok 1 - 2010 Question Paper (General medicine)
Krok 1 - 2010 Question Paper (General medicine)Eneutron
 
Krok 1 - 2014 (Histology)
Krok 1 - 2014 (Histology)Krok 1 - 2014 (Histology)
Krok 1 - 2014 (Histology)Eneutron
 
Krok 1 - 2010 Question Paper (Stomatology)
Krok 1 - 2010 Question Paper (Stomatology)Krok 1 - 2010 Question Paper (Stomatology)
Krok 1 - 2010 Question Paper (Stomatology)Eneutron
 
Krok 1 - 2014 Question Paper (Stomatology)
Krok 1 - 2014 Question Paper (Stomatology)Krok 1 - 2014 Question Paper (Stomatology)
Krok 1 - 2014 Question Paper (Stomatology)Eneutron
 

Similar to 2006 (20)

2005
20052005
2005
 
2007
20072007
2007
 
Krok 1 - 2015 (Histology)
Krok 1 - 2015 (Histology)Krok 1 - 2015 (Histology)
Krok 1 - 2015 (Histology)
 
Krok 1 - 2007 Question Paper (Stomatology)
Krok 1 - 2007 Question Paper (Stomatology)Krok 1 - 2007 Question Paper (Stomatology)
Krok 1 - 2007 Question Paper (Stomatology)
 
file
file file
file
 
Krok 1 - 2015 Question Paper (Stomatology)
Krok 1 - 2015 Question Paper (Stomatology)Krok 1 - 2015 Question Paper (Stomatology)
Krok 1 - 2015 Question Paper (Stomatology)
 
Krok 1 - 2008 Question Paper (Stomatology)
Krok 1 - 2008 Question Paper (Stomatology)Krok 1 - 2008 Question Paper (Stomatology)
Krok 1 - 2008 Question Paper (Stomatology)
 
Krok 1 - 2008 Question Paper (General medicine)
Krok 1 - 2008 Question Paper (General medicine)Krok 1 - 2008 Question Paper (General medicine)
Krok 1 - 2008 Question Paper (General medicine)
 
Krok 1 Medicine - 2016 General Medicine
Krok 1 Medicine - 2016 General MedicineKrok 1 Medicine - 2016 General Medicine
Krok 1 Medicine - 2016 General Medicine
 
Krok 1 - 2011 Question Paper (General medicine)
Krok 1 - 2011 Question Paper (General medicine)Krok 1 - 2011 Question Paper (General medicine)
Krok 1 - 2011 Question Paper (General medicine)
 
Krok 1 - 2015 (Anatomy)
Krok 1 - 2015 (Anatomy)Krok 1 - 2015 (Anatomy)
Krok 1 - 2015 (Anatomy)
 
Krok1 stomatology - 2016
Krok1   stomatology - 2016Krok1   stomatology - 2016
Krok1 stomatology - 2016
 
2009
20092009
2009
 
Krok 1 - 2014 Question Paper (General medicine)
Krok 1 - 2014 Question Paper (General medicine)Krok 1 - 2014 Question Paper (General medicine)
Krok 1 - 2014 Question Paper (General medicine)
 
Krok 1 - 2015 (Path-Physiology)
Krok 1 - 2015 (Path-Physiology)Krok 1 - 2015 (Path-Physiology)
Krok 1 - 2015 (Path-Physiology)
 
Krok 1 - 2012 Question Paper (General medicine)
Krok 1 - 2012 Question Paper (General medicine)Krok 1 - 2012 Question Paper (General medicine)
Krok 1 - 2012 Question Paper (General medicine)
 
Krok 1 - 2010 Question Paper (General medicine)
Krok 1 - 2010 Question Paper (General medicine)Krok 1 - 2010 Question Paper (General medicine)
Krok 1 - 2010 Question Paper (General medicine)
 
Krok 1 - 2014 (Histology)
Krok 1 - 2014 (Histology)Krok 1 - 2014 (Histology)
Krok 1 - 2014 (Histology)
 
Krok 1 - 2010 Question Paper (Stomatology)
Krok 1 - 2010 Question Paper (Stomatology)Krok 1 - 2010 Question Paper (Stomatology)
Krok 1 - 2010 Question Paper (Stomatology)
 
Krok 1 - 2014 Question Paper (Stomatology)
Krok 1 - 2014 Question Paper (Stomatology)Krok 1 - 2014 Question Paper (Stomatology)
Krok 1 - 2014 Question Paper (Stomatology)
 

More from Bahaa A

2013krok
2013krok2013krok
2013krokBahaa A
 
pediatric сase history
pediatric сase historypediatric сase history
pediatric сase historyBahaa A
 
Sport medicine
Sport medicineSport medicine
Sport medicineBahaa A
 
Acute pancreatitis
Acute pancreatitisAcute pancreatitis
Acute pancreatitisBahaa A
 

More from Bahaa A (7)

2013krok
2013krok2013krok
2013krok
 
pediatric сase history
pediatric сase historypediatric сase history
pediatric сase history
 
Sport medicine
Sport medicineSport medicine
Sport medicine
 
Acute pancreatitis
Acute pancreatitisAcute pancreatitis
Acute pancreatitis
 
Urology
UrologyUrology
Urology
 
Diary
Diary Diary
Diary
 
2010
20102010
2010
 

Recently uploaded

SOCIAL AND HISTORICAL CONTEXT - LFTVD.pptx
SOCIAL AND HISTORICAL CONTEXT - LFTVD.pptxSOCIAL AND HISTORICAL CONTEXT - LFTVD.pptx
SOCIAL AND HISTORICAL CONTEXT - LFTVD.pptxiammrhaywood
 
Measures of Dispersion and Variability: Range, QD, AD and SD
Measures of Dispersion and Variability: Range, QD, AD and SDMeasures of Dispersion and Variability: Range, QD, AD and SD
Measures of Dispersion and Variability: Range, QD, AD and SDThiyagu K
 
1029-Danh muc Sach Giao Khoa khoi 6.pdf
1029-Danh muc Sach Giao Khoa khoi  6.pdf1029-Danh muc Sach Giao Khoa khoi  6.pdf
1029-Danh muc Sach Giao Khoa khoi 6.pdfQucHHunhnh
 
Measures of Central Tendency: Mean, Median and Mode
Measures of Central Tendency: Mean, Median and ModeMeasures of Central Tendency: Mean, Median and Mode
Measures of Central Tendency: Mean, Median and ModeThiyagu K
 
Explore beautiful and ugly buildings. Mathematics helps us create beautiful d...
Explore beautiful and ugly buildings. Mathematics helps us create beautiful d...Explore beautiful and ugly buildings. Mathematics helps us create beautiful d...
Explore beautiful and ugly buildings. Mathematics helps us create beautiful d...christianmathematics
 
Beyond the EU: DORA and NIS 2 Directive's Global Impact
Beyond the EU: DORA and NIS 2 Directive's Global ImpactBeyond the EU: DORA and NIS 2 Directive's Global Impact
Beyond the EU: DORA and NIS 2 Directive's Global ImpactPECB
 
General AI for Medical Educators April 2024
General AI for Medical Educators April 2024General AI for Medical Educators April 2024
General AI for Medical Educators April 2024Janet Corral
 
1029 - Danh muc Sach Giao Khoa 10 . pdf
1029 -  Danh muc Sach Giao Khoa 10 . pdf1029 -  Danh muc Sach Giao Khoa 10 . pdf
1029 - Danh muc Sach Giao Khoa 10 . pdfQucHHunhnh
 
fourth grading exam for kindergarten in writing
fourth grading exam for kindergarten in writingfourth grading exam for kindergarten in writing
fourth grading exam for kindergarten in writingTeacherCyreneCayanan
 
Introduction to Nonprofit Accounting: The Basics
Introduction to Nonprofit Accounting: The BasicsIntroduction to Nonprofit Accounting: The Basics
Introduction to Nonprofit Accounting: The BasicsTechSoup
 
Paris 2024 Olympic Geographies - an activity
Paris 2024 Olympic Geographies - an activityParis 2024 Olympic Geographies - an activity
Paris 2024 Olympic Geographies - an activityGeoBlogs
 
Advanced Views - Calendar View in Odoo 17
Advanced Views - Calendar View in Odoo 17Advanced Views - Calendar View in Odoo 17
Advanced Views - Calendar View in Odoo 17Celine George
 
Disha NEET Physics Guide for classes 11 and 12.pdf
Disha NEET Physics Guide for classes 11 and 12.pdfDisha NEET Physics Guide for classes 11 and 12.pdf
Disha NEET Physics Guide for classes 11 and 12.pdfchloefrazer622
 
Student login on Anyboli platform.helpin
Student login on Anyboli platform.helpinStudent login on Anyboli platform.helpin
Student login on Anyboli platform.helpinRaunakKeshri1
 
Russian Escort Service in Delhi 11k Hotel Foreigner Russian Call Girls in Delhi
Russian Escort Service in Delhi 11k Hotel Foreigner Russian Call Girls in DelhiRussian Escort Service in Delhi 11k Hotel Foreigner Russian Call Girls in Delhi
Russian Escort Service in Delhi 11k Hotel Foreigner Russian Call Girls in Delhikauryashika82
 
Grant Readiness 101 TechSoup and Remy Consulting
Grant Readiness 101 TechSoup and Remy ConsultingGrant Readiness 101 TechSoup and Remy Consulting
Grant Readiness 101 TechSoup and Remy ConsultingTechSoup
 
Activity 01 - Artificial Culture (1).pdf
Activity 01 - Artificial Culture (1).pdfActivity 01 - Artificial Culture (1).pdf
Activity 01 - Artificial Culture (1).pdfciinovamais
 
A Critique of the Proposed National Education Policy Reform
A Critique of the Proposed National Education Policy ReformA Critique of the Proposed National Education Policy Reform
A Critique of the Proposed National Education Policy ReformChameera Dedduwage
 

Recently uploaded (20)

Mattingly "AI & Prompt Design: The Basics of Prompt Design"
Mattingly "AI & Prompt Design: The Basics of Prompt Design"Mattingly "AI & Prompt Design: The Basics of Prompt Design"
Mattingly "AI & Prompt Design: The Basics of Prompt Design"
 
SOCIAL AND HISTORICAL CONTEXT - LFTVD.pptx
SOCIAL AND HISTORICAL CONTEXT - LFTVD.pptxSOCIAL AND HISTORICAL CONTEXT - LFTVD.pptx
SOCIAL AND HISTORICAL CONTEXT - LFTVD.pptx
 
Measures of Dispersion and Variability: Range, QD, AD and SD
Measures of Dispersion and Variability: Range, QD, AD and SDMeasures of Dispersion and Variability: Range, QD, AD and SD
Measures of Dispersion and Variability: Range, QD, AD and SD
 
1029-Danh muc Sach Giao Khoa khoi 6.pdf
1029-Danh muc Sach Giao Khoa khoi  6.pdf1029-Danh muc Sach Giao Khoa khoi  6.pdf
1029-Danh muc Sach Giao Khoa khoi 6.pdf
 
Measures of Central Tendency: Mean, Median and Mode
Measures of Central Tendency: Mean, Median and ModeMeasures of Central Tendency: Mean, Median and Mode
Measures of Central Tendency: Mean, Median and Mode
 
Explore beautiful and ugly buildings. Mathematics helps us create beautiful d...
Explore beautiful and ugly buildings. Mathematics helps us create beautiful d...Explore beautiful and ugly buildings. Mathematics helps us create beautiful d...
Explore beautiful and ugly buildings. Mathematics helps us create beautiful d...
 
Beyond the EU: DORA and NIS 2 Directive's Global Impact
Beyond the EU: DORA and NIS 2 Directive's Global ImpactBeyond the EU: DORA and NIS 2 Directive's Global Impact
Beyond the EU: DORA and NIS 2 Directive's Global Impact
 
General AI for Medical Educators April 2024
General AI for Medical Educators April 2024General AI for Medical Educators April 2024
General AI for Medical Educators April 2024
 
1029 - Danh muc Sach Giao Khoa 10 . pdf
1029 -  Danh muc Sach Giao Khoa 10 . pdf1029 -  Danh muc Sach Giao Khoa 10 . pdf
1029 - Danh muc Sach Giao Khoa 10 . pdf
 
fourth grading exam for kindergarten in writing
fourth grading exam for kindergarten in writingfourth grading exam for kindergarten in writing
fourth grading exam for kindergarten in writing
 
Introduction to Nonprofit Accounting: The Basics
Introduction to Nonprofit Accounting: The BasicsIntroduction to Nonprofit Accounting: The Basics
Introduction to Nonprofit Accounting: The Basics
 
Paris 2024 Olympic Geographies - an activity
Paris 2024 Olympic Geographies - an activityParis 2024 Olympic Geographies - an activity
Paris 2024 Olympic Geographies - an activity
 
Advanced Views - Calendar View in Odoo 17
Advanced Views - Calendar View in Odoo 17Advanced Views - Calendar View in Odoo 17
Advanced Views - Calendar View in Odoo 17
 
Disha NEET Physics Guide for classes 11 and 12.pdf
Disha NEET Physics Guide for classes 11 and 12.pdfDisha NEET Physics Guide for classes 11 and 12.pdf
Disha NEET Physics Guide for classes 11 and 12.pdf
 
Student login on Anyboli platform.helpin
Student login on Anyboli platform.helpinStudent login on Anyboli platform.helpin
Student login on Anyboli platform.helpin
 
Advance Mobile Application Development class 07
Advance Mobile Application Development class 07Advance Mobile Application Development class 07
Advance Mobile Application Development class 07
 
Russian Escort Service in Delhi 11k Hotel Foreigner Russian Call Girls in Delhi
Russian Escort Service in Delhi 11k Hotel Foreigner Russian Call Girls in DelhiRussian Escort Service in Delhi 11k Hotel Foreigner Russian Call Girls in Delhi
Russian Escort Service in Delhi 11k Hotel Foreigner Russian Call Girls in Delhi
 
Grant Readiness 101 TechSoup and Remy Consulting
Grant Readiness 101 TechSoup and Remy ConsultingGrant Readiness 101 TechSoup and Remy Consulting
Grant Readiness 101 TechSoup and Remy Consulting
 
Activity 01 - Artificial Culture (1).pdf
Activity 01 - Artificial Culture (1).pdfActivity 01 - Artificial Culture (1).pdf
Activity 01 - Artificial Culture (1).pdf
 
A Critique of the Proposed National Education Policy Reform
A Critique of the Proposed National Education Policy ReformA Critique of the Proposed National Education Policy Reform
A Critique of the Proposed National Education Policy Reform
 

2006

  • 1. 1. Secretion of what gastrointestinal C. 0% hormones will be primarily decreased as a D. 100% result of iduodenum removal? E. 75% A. Histamine 7. Scraps of the mycelium of a fungus, spores, B. Gastrin and histamine air bubbles and fat drops were discovered C. Cholecystokinin and secretin on microscopy of the patient's hair excluded D. Gastrin from the infected areas. What fungus disease E. Neurotensin is characterised by this microscopic picture? 2. Low level of albumins and fibrinogen was A. Sporotrichosis detected in the patient's blood. Decreased B. Favus activity of what organelle of the liver C. Epidermophytosis hepatocytes can most probably cause it? D. Trichophytosis A. Granular endoplasmatic reticulum E. Microspory B. Lysosomes 8. From the nasopharynx of a 5-year-old child C. Golgi complex it was excreted a microorganism which D. Mitochondrions is identical to Corynebacterium diphtheriae E. Agranular endoplasmatic reticulum dose according to morphological and biochemical 3. A 10-year-old girl often experiences acute signs. But this microorganism respiratory infections with multiple punctate does not produce exotoxin. As a result haemorrages in the places of clothes friction. of what process can this microorganism Hypovitaminosis of what vitamin has the become toxigenic? girl? A. Growing with antitoxic serum A. B2 B. Phage conversion B. C C. Chromosome mutation C. B6 D. Passing through the organism of the D. B1 sensitive animals E. A E. Cultivation in the telluric environment 4. Different functional groups can be 9. A patient has undergone an amputation presented in the structure of L-amino acid's of lower extremity. Some time later painful radicals. Identify the group that is able to nodules appeared in a slump. Amputations form ester bond: neuromas were found out at the microscopic A. -OH examination. To what pathological processes B. -NH2 do those formations relate? C. -CONH2 A. Inflammation D. -CH3 B. Regeneration E. -SH C. Hyperemia 5. A 27-year-old woman has dropped penicillin D. Metaplasia containing eye drops. In a few minutes E. Dystrophy there appeared feeling of itching, burning 10. To prevent long-term effects of 4-day of the skin, lips and eyelids edema, whistling malaria a 42-year-old patient was prescribed cough, decrease of BP. What imunoglobulins primaquine. On the 3-rd day from the take part in the development of this allergic begin of treatment there appeared stomach reaction? and heart pains, dyspepsia, general cyanosis, A. IgM and IgG hemoglobinuria. What caused side effects of B. lgA and IgM the preparation? C. IgM and IgD A. Drug potentiation by other preparations D. IgG and IgD B. Cumulation of the preparation E. IgE and IgG C. Delayed urinary excretion of the 6. Hypertrychosis of auricles is caused by preparation a gene that is localized in Y-chromosome. D. Decreased activity of microsomal liver Father has this feature. What is the probability enzymes to give birth to a boy with such anomaly? E. Genetic insufficiency of glucose 6-phosphate A. 25% dehydrogenase B. 35 % 11. A patient with diabetes mellitus experienced
  • 2. loss of consciousness and convulsions after D. Balantidium injection of insulin, what is the E. Intestinal amoeba result of biochemical blood analysis for 16. A 44-year--old woman complains of concentration of the sugar? common weakness, heart pain., considerable A. 3,3 mmol/L increase of body weigt. Objectively: moonlike B. 8,0 mmol/L face, hirsutism, AP- 165/100 mm Hg. C. 5,5 mmol/L height- 164 cm, weight - 103 kg: fat is mostly D. 1,5 mmol/L accumulated in the region of neck, upper E. 10,0 mmol/L shoulder girdle, stomach. What is the main 12. A patient with complaints of dryness in pathogenetic mechanism of obesity? the mouth, photophobia and vision impairment A. Decreased production of thyroidal was admitted in the reception-room. hormones Skin is hyperemic, dry, pupils, are dilated, B. Increased production of insulin tachycardia. Poisoning with belladonna C. Decreased production of glucagon alkaloids was diagnosed on further examination. D. Increased production of glucocorticoids What medicine should be prescribed? E. Increased production of mineralocorticoids. A. Armine 17. Inflammatory process or modified B. Dipiroxim subserous layer around cervix or the uterus C. Pilocarpine caused an intensive pain syndrome. In what D. Diazepam region of genitals does the pathological E. Prozerin process take place? 13. A patient suffering from trombophlebitis of A. Endometrium the deep crural veins suddenly died. B. Perimetrium Autopsy has shown freely lying red C. Mesometrium friable masses with dim crirnped surface D. Parametrium In the trunk and bifurcation of the E. Myometrium pulmonary artery. What pathologic process 18. Microspecimen of red bone marrow was revealed by morbid anatomist? contains multiple capillares through the Ac. Thrombosis walls of which mature blood cells penetrate B. Tromboembolism into the bloodstream. What type of capillares C. Tissue embolism are these? D. Fat embolism A. Fenestrational E. Embolism with foreign body? B. Sinusoidal 14. A 38-year-old man who poisoned himself C. Somatical with mercury dichloride was taken to the D. Visceral admission room in grave condition. What E. Lymphatic antidote should be immediately introduced? 19. A patient with thrombophlebitis of A. Isonitrosine lower extremities had got chest pains, blood B. Dipiroxim spitting, growing respiratory failure that C. Unithiol caused his death. Autopsy revealed multiple D. Nalorphine pulmonary infarctions. What is the most E. Atropine probable reason of their development? 15. Slime, blood and protozoa 30-200 microns A. Pulmonary artery thrombosis long have been revealed in a man's B. Bronchial artery thrombosis feces. The body is covered with cilias and has C. Bronchial artery embolism correct oval form with a little bit narrowed D. Pulmonary venous thrombosis anterior and wide round shaped posterior E. Pulmonary artery embolism end. At the anterior end a mouth is visible. 20. An isolated cell of human heart In cytoplasm there are two nucleui and automatically generates excitation impulses two short vacuoles. What are the described with frequency 60 times pro minute. What features typical for? structure does this cell belong to? A. Lamblia A. Ventricle B. Dysenteric amoeba B. Sinoatrial node C. Trichomonas C. Atrioventricular node
  • 3. D. His' bundle B. Satellite-cells E. Atrium C. Fibroblasts 21. During examination of a patient, there D. Myofibroblasts was found a neoplasm in the white substance E. Myoblasts of cerebral hemispheres with localization in the 26. An autopsy revealed large (1-2 cm) knee and frontal part of posterior crus of brownish-red, easy crumbling formations internal capsule. Fibres of what conductive covering ulcerative defects on the external tract of' the brain will be disrupted? surface of the aortic valve. That is the most A. Tr. frontothalamicus likely diagnosis? B. Tr. parietooccipitopontinus A. Diffusive endocarditis C. Tr. pyramidalis B. Fibroplastic endocarditis D. Tr. frontopontinus C. Acute verrucous endocarditis E. Tr. thalamocorticalis D. Polypous-ulcerative endocarditis 22. A sick man with high temperature and a E. Recurrent warty endocarditis lot of tiny wounds on the body has been 27. A patient died from acute cardiac admitted to the hospital. Lice have been insufficiency. The histological examination found in the folds of his clothing. What disease of his heart revealed the necrotized section can be suspected? in myocardium of the left ventricle, which A. Plague was separated from undamaged tissue B. Epidemic typhus by the zone of hyperimic vessels, small C. Malaria hemorrhages and leukocytic infiltration. D. Tularemia What is the most likely diagnosis? E. Scabies A. Diffuse exudate myocarditis 23. Autopsy of a 46-year-old man revealed B. Productive myocarditis Multiple brown-and-green layers and C. Focal exudate myocarditis hemmorhages on the mucous membrane D. Myocardial ischemic dystrophy of rectum and sigmoid colon; slime and E. Myocardial infarction some blood in colon lumen; histologically 28. A patient operated on complicated - fibrinous colitis. In course of bacteriological appendicitis has the following changes of analysis of colon contents S.Sonne were blood count: erythrocytes – 4,0 ·1012 /1, found. What is the most probable diagnosis? Hb - 120 gll. color index - 0.9, leukocytes A. Crohn’s disease - 18·109 /1, basophils - 0, eosinophils - 0, B. Cholera myelocytes - 0, juvenile - 0, stab neutrophils C. Salmonellosis - 20, segmentonuclear neutrophils - 53, D. Dysentery lymphocytes - 21, monocytes - 5. How is such E. Yersiniosis nuclear shift of leukocy1ic formula called? 24. Intake of oral contraceptives containing A. Degenerative left shift sex hormones inhibits secretion of the B. Regeneratively- degenerative hypophysiae hormones. Secretion of which C. Hyperregenerative of the indicated hormones is inhibited D. Right shift while taking oral contraceptives with sex E. Regenerative left shift hormones? 29. A tissue sample of benign tumor was A. Somatotropic studied under the electron microscope. A lot B. Oxytocin of small (15-20 nm) spherical bodies, consisting C. Vasopressin of 2 unequal subunits were detected. D. Follicle-stimulating These are: E. Thyrotropic A. Mitochondria 25. Patient with injured muscles of the B. Smooth endoplasmic reticulum lower extremities was admitted to the C. Ribosomes traumatological department. Due to what D. Golgi complex cells is reparative regeneration of the muscle E. Microtubules fibers and restoration of the muscle function 30. A 45-year-old man applied to the possible? trauma station because of domestic shoulder A. Myoepithelial cells trauma. Objectively: extension, reduction
  • 4. and pronation functions of the shoulder are dystrophic changes of parenchymatous absent. What muscle was injured? organs, brain edema. What complication can A. Supraspinous muscle cause such changes of serous membranes B. Subscapular muscle and inner organs? C. Infraspinous muscle A. Thrombopenia D. Teres minor muscle B. DIC-syndrome E. Teres major muscle C. Anemia 31. Kidneys of a man under examination D. Uraemia show increased resorbtion of calcium ions and E. Sepsis decreased resorbtion of phosphate ions. 36. In course of prophylactic medical What hormone causes this phenomenon? examination 7-year-old boy was A. Parathormone diagnosed to have daltonism. B. Hormonal form D3 Parents are healthy color C. Aldosterone vision is normal. But grandfather from the D. Thyrocalcitonin mother’s side has the same disorder. E. Vasopressin What is the type of inheriting of this anomaly? 32. Increased fragility of vessels, enamel and A. Incomplete domination dentine destruction resulting from scurvy B. Autosomal - dominant are caused by disorder of collagen maturation. C. Recessive, sex-linked What stage of procollagen modification D. Dominant, sex-linked is disturbed under this avitaminosis? E. Autosomal - recessive A. Hydroxyla1ion of proline 37. A patient with encephalopathy B. Formation of polypeptide chains was admitted to the neurological inpatient C. Detaching of N-ended peptide department. There was revealed D. Removal of C-ended peptide from a correlation between increasing of procollagen encephalopathy and substances absorbed by E. Glycosylation of hydroxylysine residues the bloodstream from the intestines. What 33. A 7-year-old girl has signs of anemia. substances that are formed in the intestines Laboratory examination revealed pyruvate can cause endotoxemia? kinase deficiency in erythrocytes. What A. Biotin process disturbance plays· the main role in B. Indole anemia deve1opment? C. Acetacetate A. Oxidative phosphorylation D. Butyrate B. Anaerobic glycolysis E. Ornithine C. Tissue respiration 38. A patient with hypochromic anemia has D. Peroxide decomposition splitting hair and loss of hair, increased nail E. Aminoacids desamination brittling and taste alteration. What is the 34. During the endoscopy the inflammation mechanism of the development of these of a major papilla of the duodenum and the symptoms? disturbances of bile secretion were found. In A. Decreased production of thyroid which part of duodenum were the problems hormones found? B. Deficiency of vitamin A A. Upper horizontal part C. Decreased production of parathyrin B. Ascendant part D. Deficiency of vitamin B12 C. Bulb E. Deficiency of iron-containing enzymes D. Lower horizontal part 39. A patient visited a dentist with complaints E. Descendent part of redness and edema of his mouth 35. For a long time a 49-year-old woman mucous membrane in a month after dental had suffered from glomerulonephritis which prosthesis. The patient was diagnosed with caused death. The autopsy revealed that the allergic stornatitis. What type of allergic size of her kidney's was 7 x 3 x 2,5 cm, weight reaction by Gell and Cumbs underlies this 65, 0 g, they were dense and small-grained. disease? Microscopically fibrinogenous inflammlation A. Cytotoxic of serous and mucous membranes, B. Delayed type hypersensitivity
  • 5. C. Stimulating permeability of vessels of microcirculation D. Immunocomplex stream, increase of their hydrodynamic E. Anaphylactic blood pressure. Increasing of the osmotic 40. Most participants of Magellan expedition concentration and dispersity of protein to America died from avitominosis. This structures is present in the intercellular fluid. disease declared itself by general weakness, What kind of edema will appear in this case? subcutaneous hemorrhages, falling of A. Colloid-osmotic teeth, gingival hemorrhages. What is the B. Lymphogenic name of this avitominosis? C. Mixed A. Polyneuritis (beriberi) D. Hydrodynamic B. Pellagra E. Membranogenic C. Rachitis 46. A 46-year-old man complains of difficulties D. Scurvy with nasal breathing. Mikulicz's E. Biermer's anemia cells, accumulation of epithelioid cells, 41. A 17-year-old boy fell seriously ill body plasmocytes, lymphocytes; hyaline balls temperature rose up to 38,5°C, there were discovered in the biopsy material of the is cough, rhinitis, lacrimation, nasal discharges. thickened nasal mucosa. What is the most likely What kind of inflammation is it? diagnosis? A. Suppurative inflammation A. Meningococcal nasopharyngitis B. Hemorrhagic inflammation B. Rhinovirus infection C. Serous inflammation C. Virus rhinitis D. Fibrinous inflammation D. Scleroma E. Catarrhal inflammation E. Allergic rhinitis 42. The donor who didn't donate the blood 47. A 60-year-old patient fell ill with for a long time was investigated with obturative jaundice as a result of malignant IFA method. Anti-HBs antibodies were tumour of the big papillary of the duodenal. revealed. What does positive result of IFA Lumen of what anatomical structure is in this case mean? compressed by tumour? A. Previous hepatitis B A. Left hepatic duct B. Chronic hepatitis C B. Common hepatic duct C. Chronic hepatitis B C. Right hepatic duct D. Acute hepatitis B D. Hepatopancreatic ampulla E. Acute hepatitis C E. Cystic duct 43. A 42-year-old man suffering from gout 48. Desulfiram is widely used in medical has increased level of urinary acid in blood. practice to prevent alcocholism. It inhibits Allopurinol was prescribed to decrease the aldehyde dehydrogenase. Increased level of urinary acid. Competitive inhibitor level of what metabolite causes aversion to of what enzyme is allopurinol? alcochol? A.Xanthine oxidase A. Propionic aldehyde B. Guanine deaminase B. Methanol C. Adenosine deaminase D. Ethanol D. Hypoxanthine phosphoribosiltransferase D. Acetaldehyde E. Adenine phosphoribosiltransferase E. Malonyl aldehyde 44. While preparing a patient to the operation 49. An experiment proved that UV-radiated the heart chambers' pressure was cells of patients with xeroderma measured. In one of them the pressure pigmentosum restore the native DNA changed during one heart cycle from 0 to structure slower than cells of healthy individuals 120 mm Hg. What chamber of heart was it? as a result of reparation enzyme A. Left atrium defection. What enzyme helps this process? B. Left ventricle A. Primase C. Right ventricle B. DNA polymerase III D. - C. DNA gyirase E. Right atrium D. RNA ligase 45. Inflammation is characterised bv increased E. Endonuclease
  • 6. 50. An old woman was hospitalized with A. NK-cells acute pain, edema in the right hip joint; the B. Macrophages, monocytes movements in the joint are limited. Which C. 0-lymphocytes bone or part of it was broken? D. T-lymphocytes, B-lymphocytes A. The body of the thigh bone E. Fibroblasts, T-lymphocytes, B-lymphocytes B. Ischial bone 55. The effect of electric current on the C. Pubic bone exitable cell caused depolarization of its D. Condyle of the thigh membrane. Movement of what ions through E. The neck of the thigh the membrane caused depolarization? 51. In the blood of a 26-year-old man A. HCO3- 18% of erythrocytes of the spherical, B. Ca2+ ballshaped, flat and spinous shape have been C. K+ revealed. Other erythrocytes were in form D. Cl- of the concavo-concave disks. How is this E. Na+ phenomenon called? 56. A patient with diabetes mellitus has been A. Pathological anisocytosis delivered in hospital in the state of B. Pathological poikilocytosis unconsciousness. Arterial pressure is low. C. Erytrocytosis The patient has acidosis. Point substances, D. Physiological poikilocytosis which accumulation in the blood results in these E. Physiological anisocytosis manifestations: 52. Testosterone and it’s analogs increase A. High fatty acids the mass of skeletal muscles that allows to B. Amino acids use them for treatment of dystrophy. Due to C. Ketone bodies interaction of the hormone with what cell D. Cholesterol esters substrate is this action caused? E. Monosaccharides A. Membrane receptors 57. Synthesis of phospholipids is disturbed B. Chromatin as a result fatty infiltration of liver. Indicate C. Ribosomes which of the following substances can D. Nuclear receptors enhance the process of methylation during E. Proteins-activators of transcription phospholipids synthesis? 53. A 39-vear-old woman has madescence in A. Ascorbic acid the region or mammilla, a small ulcer with B. Glucose inflammatory hyperemia and cutaneous C. Methionine edema. Histologic examination of tissue D. Glycerin sampling from this area revealed in the E. Citrate malpighian layer of thickened epidermis 58. Donor skin transplantation was atypical cells with light and optically empty performed to a patient with extensive burns. cytoplasm. With no intracellular bridges. On the 8-th day the graft became swollen Such cells were also found in the orifice of and changed color; on the 11-th day graft big mammal gland ductus. What is the most rejection started. What cells take part in this probable diagnosis? process? A. Epidermoid cancer A. B-lymphocytes B. Melanocarcinoma B. Erythrocytes C. Basal cell carcinoma C. Basophils D. Intraductal cancer D. T-lymphocytes - E. Paget’s disease E. Eosinophils 54. A patient with clinical signs of 59. A 38-year-old patient died during immunodeficiency has no changes of intractable attack of bronchial asthma. the number and functional activity of Histologic examination revealed mucus T- and B- lymphocytes. Defect with dysfunction accumulation in bronchial lumen, a lot of of antigen-presentation to the fat cells (labrocytes) in the wall of bronches, immunocompetent cells was found during many of them are in the state of degranulation, examinatio on the molecule level. Defect of there are also a lot of eosinophils. What what cells is the most probable? pathogenesis of bronchial changes is it?
  • 7. A. Granulomatosis C. Pus in the wound B. Immunocomplex mechanism D. Low concentration H2O2 C. Cellular cytolysis E. Inherited insufficiency of catalase D. Atopy 65. A patient with continious bronchopneumonia E. Cytotoxic, cytolytic action of antibodies was admitted to the therapeutic department. 60. A 2-year-old child has got intestinal Antibiotic therapy didn't give much effect. dysbacteriosis, which resultsed in What medication for improvement of hemorrhagic syndrome. What is the most immune state should be added to the likely cause of hemorrhage of the child? complex treatment of this patient? A. Fibrinogen deficiency A. Timaline B. Hypocalcemia B. Analgin C. PP hypovitaminosis C. Sulfocamphocaine D. Vitamin K deficiency D. Paracetamol E. Activation of tissue thromboplastin E. Benadryl 61. Autopsy revealed that right lung is 66. Decreased blood supply to the organs enlarged, solid, there are fibrin layers on causes hypoxia that activates fibroblasts the pleura. Lung tissue is light grey color on function. Volume of what elements is incision with muddy liqued exudates.What increased in this case? lung disease are these symptoms typical for? A. Parenchymatous elements of an organ A. Interstitial pneumonia B. Vessels of microcircular stream B. Bronchopneumonia C. Lymphatic vessels C. Croupous pneumonia D. Intercellular substance D. Pulmonary gangrene E. Nerve elements E. Fibrosing alveolilis 67. A doctor administered Allopurinol to a 62. Blood analysis of a patient showed signs 26-year-old young man with the symptoms of HIV infection (human immunodeficiency of gout. What pharmacological action of virus). Which cells does HIV-virus primarily Allopurinol ensures therapeutical effect? affect? A. By inhibiting leucocyte migration into the joint A. Proliferating cells (stem hematoplastic B. By general analgetic effect cells) C. By increasing uric acid excretion B. Specialized nervous cells (neurons) D. By general anti-inflammatory effect C. Cells that contain receptor IgM E. By inhibiting uric acid synthesis (B-lymphocytes) 68. Thyrotoxicosis leads to increased D. Mast cells production of thyroidal hormones T3 and E. Cells that contain receptor T4 (T-helpers) T1, weight loss, tachycardia, psychic 63. A 60-year-old patient has reduced excitementand so on. How do thyroidal perception of high-frequency sounds. What hormones effect energy metabolism in the structures' disorder of auditory analyzer mitochondrion of cells? caused these changes? A. Disconnect oxidation and oxidative A. Eustachian tube phosphorylation B. Main membrane of cochlea near the oval B. Stop respiratory chain window C. Activate oxidative phosphorylation C. Muscles of middle car D. Stop substrate phosphorylation. D. Main membrane of cochlea near helicotrerna E. Activate substrate phosphorylation E. Tympanic membrane 69. A patient with rheumatoid arthritis who 64. Patient with abscess of the cut wound had been treated with indometacin has got applied to the traumatological department. signs of gastropathy.thy. What activity of the In order to clean the wound from the pus drug can this complication be connected with? doctor washed it with 3% hydrogen peroxide. A. Anticyclooxygenase Foam was absent. What caused the B. Antiserotonin absence of the drug activity? C. Antihistamine A. Shallow wound D. Locally irritating R. Inherited insufficiency of erythrocyte E. Antikinine phosphatdehydrogenase. 70. Dystrophic alterations of heart are
  • 8. accompanied with dilation of heart cavities, A. Parathormone decreased force of heart contractions, B. Aldosterone increased blood volume that remains during C. Adrenalin systole in the heart cavity, vein overfill. D. Vasopressin What heart condition is it typiccal for? E. Auricular sodiumuretic factor A. Myogenic dilatation 76. Intrapleural pressure of an individual is B. Cardiac tamponade being measured. In what phase did he hold C. Emergency stage of hyperfunction and his breath if the pressure is - 25 cm H2O? hypertrophy A.- D. Cardiosclerosis E. Forced expiration E. Tonogenic dilatation C. Forced inspiration 71. A physician examined a patient and D. Quiet expiration found inguinal hernia. Through what E. Quiet inspiration anatomic formation does it penetrate into 77. A 37-year-old man was admitted the skin? to the surgical department with the A. Lacuna musculorum symptoms of acute pancreatitis: vomiting, B. Anulus inguinalis super ficialis diarrhea, bradycardia, hypotention. C. Canalis adductorius weakness, dehydration of the organism. D. Anulus femoralis What medicine should be used first of all? E. Hiatus saphenus A. No-spa 72. A 60-year-old man felt asleep after B. Etaperazine cerebral hemorrhage for a long time. C. Ephedrine Damage of what structure caused this state? D. Platyphylline A. Cortex of the large hemispheres E. Contrycal B. Hippocampus 78. The alternate usage or dichlotiazide, C. Nuclears of the cerebral nerves etacrin acid and lasex did not influence D. Reticular formation diuretically upon the patient with marked E. Black substance peripheral edemata. The aldosterone rate in 73. A 25-year-old woman with red and itchy the blood is increased. Indicate which medicine eczematoid dermatitis visits your office. She should be prescribed: had a dental procedure one day earlier with A. Urea administration of a local anesthetic. There B. Amilorid were no other findings, although she indicated C. Clopamid that she had a history of allergic reactions. D. Spironolacton Which of the following drugs is most likely E. Mannit involved? 79. A person has steady HR not exceeding A. Lidocaine 40 bpm. What is the pacemaker of this B. Cocaine person's heart rhythm? C. Bupivacaine A. Branches of His' bundle D. Procaine B. Purkinje's fibers E. Etidocaine C. Atrioventricular node 74. A patient with suspicion on epidemic D. Sinoatrial node typhus was admitted to the hospital. Some E. His' bundle arachnids and insects have been found in his 80. A 34-year-old patient was diagnosed with flat. Which , of them, may be a carrier of the chronic glomerulonephritis 3 years ago. pathogen or epidemic typhus? Edemata have developed within the last 6 A. Spiders month’s. What caused the edemata? B. Cockroaches A. Hyperosmolarity of plasma C. Bed-bugs B. Hyperproduction of vasopressin D. Lice C. Proteinuria E. Houseflies D. Liver dysfunction of protein formation 75. A person has reduced diuresis, E. Hyperaldosteronism hypernatremia, hypokalemia. Hypersecretion 81. A 5-year-old child who often fells ill of what hormone can cause such changes? with respiratory diseases has eczematous
  • 9. appearances after consumption of some hyperemia and skin edema in few hours food products, tendency to prolonged after burn. What mechanism strengthens course of inflammatory processes. What kind destructive effect in the inflammation area? of diathesis can be suspected in this case? A. Diapedesis of erythrocytes A. Asthenic B. Proliferation of fibroblasts B. Lymphohypoplastic C. Primary alteration C. Arthritism D. Emigration of lymphocytes D. Exudative-catharral E. Secondary alteration E. Hemmorhagic 87. A patient's blood was analyzed and 82. An 18-year-old patient has enlarged the decreased erythrocyte's sedimentation inguinal lymph nodes, they are painless, rate (ESR) was discovered. What disease thickened on palpation. In the area of genital from the listed below is accompanied with mucous membrane there is a small sized decreased ESR? ulcer with thickened edges and A. Polycytemia “laquer” bottom of greyish color. What is B. Myocardial infarction the most probable diagnosis? C. Vitamin B deficiency A. Syphilis D. Hepatitis B. Trophic ulcer E. Splenomegaly C. Tuberculosis 88. A considerable increase of activity of D. Gonorrhea MB-forms of CPK (creatinephosphokinase) E. Lepra and LDH-1 was revealed on the examination 83. A 56-year-old patient complaining of thirst of patient's blood. What is the most likely and frequent urination was diagnosed with pathology? diabete mellitus. Butamin was prescribed. A. Pancreatitis How does the medicine act? B. Rheumatism A. It relieves transport of glucose through C. Hepatitis the cells' membranes D. Myocardial infarction B. It stimulates β-cells of Langergans' islets E. Cholecystitis C. It helps to absorb the glucose by the cells 89. A patient had been ill with bronchial of the organism tissues asthma for many years and died from D. It inhibits α-cells of Langergans" islets asthmatic fit. Histologic lung examination E. It inhibits absorption of glucose in the revealed: lumen of bronchioles and small intestines bronches contain a lot of mucus with some 84. A l-year-old child with symptoms of eosinophils, there is sclerosis of alveolar muscle involvement was admitted to the septums, dilatation of alveole lumen. What hospital. Examination revealed carnitine mechanism of development of hypersensibility deficiency in his muscles. What process reaction took place? disturbance is the biochemical basis of this A. Cytolysis determined by lymphocytes pathology? B. Cytotoxic reaction A. Actin and myosin synthesis C. Granulomatosis B. Regulation of Ca2+ level in mitochondrion’s D. Immunocomplex reaction C. Substrate phosphorylation E. Reagin reaction - D. Lactic acid utilization 90. The process of heart transplantation E. Transporting of fatty acids to mitochondrion’s determined the viability of myocardlial cells. 85. A patient with tissue trauma was taken The determination of what myocardium a blood sample for the determination of parameter is the most important? blood clotting parameters. Specify the right A. Rest potential of cardiomyocytes sequence of extrinsic pathway activation: B. Heart temperature A. IV - VIIa - Xa C. Concentration of Ca-ions in heart vessels B. IV - VIII: TF - Xa D. Concentration of oxygen in heart vessels C. III - VIII: TF - Xa E. Concentration of calcium-ions in myofibrils D. III - IV - Xa 91. Part of the DNA chain turned 180 degree E. III - VIIa - Xa as a result of gamma radiation. What type of 86. Necrosis focus appeared in the area of mutation took place in the DNA chain?
  • 10. A. Doubling C. Autosomal dominant B. Deletion D. X-linked dominant C. Translocation E. X-linked recessive D. Replication 97. A 45-year-old woman suffers from E. Inversion seasonal allergic rhinitis caused by the 92. An individual is characterized by ambrosia blossoming. What medicine from rounded face, broad forehead, a mongolian the stabilizer of the adipose cells group can type of eyelid fold, flattened nasal c bridge, be used for prevention of this disease? permanently open mouth, projecting A. Diazoline lower lip, protruding tongue, short neck, flat B. Tavegyl hands, and stubby fingers. What diagnosis C. Phencarol can be put to the patient? D. Dimedrol A. Klinefelter's syndrome E. Ketotifen B. Supermales 98. After a serious viral infection a 3-year-old C. Alkaptonuria child has, repeated vomiting, loss D. Down’s syndrome of consciousness, convulsions. Examination E. Turner's syndrome revealed hyperammoniemia. What may 93. Autopsy of a man who died from have caused changes of biochemical blood chronic cardiovascular collapse revealed indices of this child? "tiger heart". Sidewards of endocardium A. Inhibited activity of transamination a yellowish-white banding can be seen; enzymes myocardium is dull, dark-yellow. What B. Disorder of ammonia neutralization in process caused this pathology? ornithinic cycle A. Carbohydrate degeneration C. Disorder of biogenic amines neutralization B. Fatty parenchymatous degeneration D. Activated processes of aminoacids C. Amyloidosis decarboxylation D. Hyaline degeneration E. Increased putrefaction of proteins in E. Fatty vascular-stromal degeneratioin intestines 94. Children often have heavy nasal breathing 99. Part of alveoles of a preterm infant didn't resulting from excessive development spread because of enhanced elastic recoil of of lymphoid tissue of pharyngeal mucous lungs. How can this recoil be reduced? membrane. What tonsils growth may cause A. By fluid suction from the respiratory tracts this effect? B. By glycose introduction A. All above mentioned tonsils C. By surfactant introduction B. Tonsilla palatina D. By artificial pulmonary ventilation C. Tonsilla pharyngea E. By pure oxygene inhalation D. Tonsilla tubaria 100. A mother of a newborn complains E. Tonsilla lingualis of her baby's constant belching with undigested 95. ATP synthesis is totaly blocked in a milk. Which developmental anomaly cell. How will the value of membrane rest is it an evidence of? potential change? A. Faux lupinum A. It will disappear B. Esophageal atresia B. First it will decrease, then increase C. Labium leporiurn C. First it will increase, then decrease D. Anal atresia D. It will be considerably increased E. Esophageal fistula E. It will be slightly increased 101. During the operation on the hip joint of 96. A genetics specialist analyzed the a 5-year-old child her ligament was damaged genealogy of a family and found that both which caused bleeding. What ligament was males and females may have the illness, not damaged? across all the generations, and that healthy A. Perpendicular of the acetabule parents may have ill children. What is the B. Iliofemoral type of illness inheritance? C. Pubofemoral A. Y-linked D. Ischiofemoral B. Autosomal recessive E. The head of the thigh
  • 11. 102. When a patient with traumatic impairment E. Rifampicin of the brain was examined, it was discovered 107. To anaesthetize the surgical treatment that he had stopped to distinguish of burn surface, a patient was intravenously displacement of an object on the skin. What injected a medication for short-acting part of the brain was damaged? narcosis. 1 minute later the patient being A. Frontal zone under anaesthesia showed increased blood B. Occipital zone of the cortex pressure, tachycardia, increased tone of C. Frontal central gurus skeletal muscles: reflexes remained. After D. Posterior central gurus recovering from anaesthesia the patient E. Parietal zone of the cortex had disorientation and visual hallucinations. 103. While having the dinner the child What medication was the patient injected? choked and aspirated the food. Meavy A. Ketamine cough has started, skin and mucose are B. Nitrous oxide cyanotic, pulse is rapid, respiration is C. Diethyl ether infrequent, expiration is prolonged. What D. Sombrevin disorder of the external respiration has the child? E. Thiopental sodium A Stenotic respiration 108. Healthy parents have got a fair-haired, B. Stage of inspiratory dyspnea on asphyxia blue-eyed girl. Irritability, anxiety, sleep and C. Stage of expiratory dyspnea on asphyxia feeding disturbance developed in the first D. Biot's respiration months of the infant's life. Neurological E. Alternating respiration examination revealed developmental 104. The ovary specimen colored with lag. What method of genetic investigation hematoxylin-eosin contains a follicle, in which should be used for the exact diagnosis? cubic-shaped follicle epithelium cells are A. Gemellary placed in 1-2 layers, and scarlet membrane is B. Biochemical seen around the ovocyte. Name this follicle: C. Population-statistical A. Atretic D. Genealogical B. Primary E. Cytological C. Secondary 109. A large-scale reaction with D. Mature parapertussis and pertussis diagnosticums E. Primordial was made in order to make serological 105. A 58-year-old patient with acute cardiac diagnostics of the whooping cough. At the insufficiency has decreased volume of daily bottom of the test-tubes with diagnosticum urine - oliguria. What is the mechanism of of Bordetella parapertussis a granular this phenomenon? sediment formed. What antibodies did this A. Decreased number of functioning reaction reveal? glomerules A. Antitoxins B. Drop of oncotic blood pressure B. Precipitins C. Rise of hydrostatic blood pressure 111 C. Opsonins capillars D. Agglutinins D. Reduced permeamility of renal filter E. Bacteriolysins E. Decreased glomerular filtration 110. A patient, who suffers from congenital 106. A 35-year-old man under the treatment erythropoietic porphyria, has skin for pulmonary tuberculosis has acute pain photosensitivity. The accumulation of what onset of right big toe, swelling and lowgrade compound in the skin cells can cause it? fever. The gouty arthritis was diagnosed A. Uroporphyrinogen 1 and high serum uric acid level B. Heme was found. Which of the following C. Uroporphyrinogen 2 antituberculous drugs are known for D. Protoporphyrin causing high uric acid levels? E. Coproporphyrinogen 3 A. Pyrazinamide 111. As a result of the damage of one of B. Thiacetazone the Atomic Power Plant reactors the runout C. Aminosalicylic acid of radioelements took place. People D. Cycloserine in the high-radiation area were radiated
  • 12. with approximately 250-300 r. They were have been blocked on an isolated rabbit’s immediately hospitalized. What changes in heart. What changes in the heart's activity the blood count would be typical for the victims? can result from it? A. Anemia A. Heart stops in diastole B. Lymphopenia B. Heart stops in systole C. Thrombopenia C. Decreased hear beat rate D. Neutropenia D. Decreased rate and force of heart beat E. Leukopenia E. Decreased force of the contraction 112. The pulmonalis embolism has suddenly 118. Objective examination of a patient developed in a 40-year-old patient with revealed: slender figure, big skull, highly opened fracture of the hip. Choose the possible developed frontal region of face, short kind of embolism: extremities. What constitutional type is it A. Fat characteristic for? B. Thrombus-embolus A. Muscular C. Air B. Respiratory D. Foreign body C. Cerebral E. Tissue D. Digestive 113. Nowadays about 50 minor bases have E. Mixed been found in the t-RNA structure besides 119. Blood sampling for the haematology the main four nitrogenous bases. Choose the is recommended to carry out on an empty minor nitrogenous base: stomach and in the morning. What changes A. Cysteine in blood formula are possible if blood B. Dihydrouracil sampling was carried out after food intake? C. Uracil A. Increase of erythrocyte number D. Cytosine B. Increase of plasm proteins E. Adenine C. Increase of leukocyte number 114. A patient has elbow joint trauma with D. Decrease of thrombocyte number avulsion of medial epicondyle of humerus. E. Decrease of erythrocyte number What nerve can be damaged in this trauma? 120. 40-year-old patient complains of A. Medial cutaneous nerve of forearm intensive heartbeats, sweating, nausea, visual B. Cardiac cutaneous nerve impairment, arm tremor, hypertension. C. Musculocutaneous nerve From his anamnesis: 2 years ago he D. Radial was diagnosed with pheochromocytoma. E. Ulnar Hyperproduction of what hormones causes 115. Diabetes mellitus causes ketosis as a the given pathology? result of activated oxidation of fatty acids. A. Catecholamines What disorders of acid-base equilibrium B. Aldosterone may be caused by excessive accumulation of C. Glucocorticoids ketone bodies in blood? D. Thyroidal hormones A. Metabolic alkalosis E. ACTH B. Metabolic acidosis 121. Microspecimen of spinal cord contains a C. Any changes woun't happen nucleus that should be analyzed. lts D. Respiratory acidosis neurons form motor endings in the skeletal E. Respiratory alkalosis muscles. What nucleus of spinal cord is 116. A 30-vear-old woman was diagnosed meant? with insufficiency of exocrinous function of A. Proper nucleus of the posterior horn pancreas. Hydrolisis of what nutrients will B. Intermediate lateral nucleus be disturbed? C. Thoracic nucleus A. Proteins D. Proper nucleus of gray substance B. Fats, carbohydrates E. Proper nucleus of the anterior horn C. Proteins, carbohydrates 122. Parents with an ill child consulted D. Proteins. fats an infectionist. They had been working in E. Proteins, fats, carbohydrates one of Asian countries for a long time. 117. The calcium canals of cardiomyocytes The child has sallow skin, loss of appetite,
  • 13. laxity, enlarged liver, spleen, peripheral B. Uric acid lymph nodes. What protozoal illness can be C. Glucose suspected? D. Common lipids A. Balantidiasis E. Mineral salts B. Lambliasis 128. A 50-year-old patient was injured on C. Toxoplasmosis the occipital region of the head. The dosed D. Visceral leishmaniasis skull trauma was diagnosed. She was taken E. Amebiasis to the hospital. The medical examination: 123. In the specimen of one of the parts deregulation of walking and balance. of respiratory system a tubular organ was trembling of arms. What part of brain was found. It has low epithelium, well developed injured? muscular tunic, glands and cartilage are A. The spinal cord absent. Name this organ: B. The mind-brain A. Minor bronchs C. The inter-brain B. Major bronchs D. The cerebellum C. Median bronchs E. The medulla oblongata D. Trachea 129. A woman with 0 (l) blood group E. Larynx has born a child with AB blood group. 124. A 63-year-old woman shows symptoms This woman's husband has A blood group. of rheumaioid arthritis. The increase of what What genetic interaction explains this blood indices could bc the most significant phenomenon? for proving the diagnosis? A. Polymery A. Additive glycosaminoglycans B. Recessive epistasis B. Lipoproteids C. Codominance C. Acid phosphatase D. Incomplete dominance D. R-glycosidase E. Complementation E. General cholesterol 130. Patients with bile ducts obstruction 125. A person felt thirsty after staying under suffer from inhibition of blood coagulation, the conditions of hot weather for a long time. bleedings as a result of low level of vitamin Signals of what receptors caused it first assimilation. What vitamin is in deficiency? of all? A. E A. Osmoreceptors of liver B. D B. Glucoreceptors of hypothalamus C. K C. Osmoreceptors of hypothalamus D. A D. Baroreceptors of aortic arch E. Carotene E. Sodium receptors of hypothalamus 131. A businessman came to India from 126. A 57-year-old patient was admitted South America. On examination the physician to the gastroenterological department with found that the patient was suffering suspicion of Zollinger- Ellison syndrom from sleeping-sickness.What was the way of because of rapid increase of gastrin level invasion? in the blood scrum. What is the most A. As a result of mosquito`s bites probable disorder of the secretory function B. Through dirty hands of stomach? C. With contaminated fruits and vegetables A. Hyperacidic hyposecretion D. After contact with sick dogs B. Hypoacidic hyposecretion E. As a result of bug's bites C. Hyperacidic hypersecretion 132. Examination of a newborn boy's genitals D. Hypoacidic hypersecretion revealed a cleft of urethra that opens E. Achylia on the inferior surface of his penis. What 127. A patient with serious damage of developmental anomaly is meant? muscular tissue was admitted to the A. Monorchism traumatological department. What biochemical B. Hypospadia urine index will be increased in C. Epispadia this case? D. Cryptorchism A. Creatinine E. Hermaphroditism
  • 14. 133. A pregnant woman had been having performed to reveal latent toxoplasmosis toxicosis with severe repeated vomiting and chronic gonorrhoea of the patient? for 24 hours. In the end of the day there A.. IFA - Immunofluorescence assay appeared tetanic convulsions and fluid loss. B. (R)CFT- Reiter's complement fixation What shift of acid-base state caused these test changes? C. RTHA - Reverse indirect hemagglutination A. Excretory alkalosis assay B. Excretory acidosis D. Immunoblot analysis C. Metabolic acidosis E. RDHA - Reverse direct hemagglutination D. Gaseous alkalosis assay E. Gaseous acidosis 139. A 16-year-old boy was performed an 134. Live vaccine was injected into the appendectomy. He has been hospitalized human body. Increasing activity of what for right lower quadrant abdominal pain connective tissue cells can be expected? within 18 hours. The surgical specimen is A. Adipocytes and adventitious cells edematous and erythematous. Infiltration B. Macrophages and fibroblasts by what of the following ceils is the most C. Fibroblasts and labrocytes typical for the process occurring here? D. Pigmentocytes and pericytes A. Monocytes E. Plasmocytes and lymphocytes B. Limphocytes 135. On autopsy of the 58-year-old man it C. Basophils was revealed: mitral valve is deformed, D. Eosinophils thickened, not totally closed. Microscopically: E. Neutrophils foci of collagen fibers are eosinophilic, have 140. A patient who has been treated positive fibrin reaction. The most likely it is: with diazepam on account of neurosis A. Fibrinoid swelling complains of toothache. Doctor administered B. Amyloidosis him an analgetic, but its dose was C. Mucoid swelling lower then average therapeutic dose. What D. Hyalinosis phenomenon did the doctor take into E. Fibrinoid inflammation account while prescribing the patient an 136. A 38-year-old woman was admitted to underdose? the admission-diagnostic department with A. Drug dependence uterine bleeding. What are the most likely B. Cumulation changes of blood? C. Tolerance A. Leukopenia D. Summation B. Leucocytosis E. Potentation C. Reduction of haematocrite rate 141. Glomerular filtration rate (GFR) D. Increase of haematocrite rate increased by 20% as a result of prolonged E. Polycythemia starvation of an individual. The most evident 137. A 60-year-old patient was admitted to cause of filtration changes under this the surgical department because of infection conditions is: caused by blue pus bacillus (Pseudomonas A. Increase of filtration coefficient aeruginosa) which is sensitive to penicillin B. Increase of systemic blood pressure antibiotics. Indicate which of the given C. Decrease of oncotic pressure of blood penicillin’s has marked activity to the plasma Pseudomonas aeruginosa? D. Increase of penetration of the renal filter A. Methicillin E. Increase of renal plasma stream B. Phenoxymethylpenicillin 142. The preventive radio protector was given C. Carbenicillin disodium to worker of a nuclear power station. D. Oxacillin What mechanism from the below mentioned E. Benzylpeniciilin is considered to be the male mechanism of 138. A patient who came to the doctor radioprotection? because of his infertility was administered A. Prevention of tissue's hypoxia to make tests for toxoplasmosis and chronic B. Inhibition of free radicals formation gonorrhoea. Which reaction should be C. Activation of oxidation reactions
  • 15. D. Increasing of tissue blood supply bar. Objectively: pain while moving upper E. Increasing of respiration extremity, reduced pronation and adduction 143. Examination of initial molecular functions. Sprain of what muscle can be structure revealed substitution of the observed here? glutamic acid by valine. What inherited A. M. latissimus dorsi pathology is it typical for? B. M. subscapularis A. Hemoglobinosis C. M. romboideus major B. Thalassemia D. M. levator scapulae C. Minkowsky-Shauffard disease E. M. trapezius D. Favism 149. A young man has a painlessl formation E. Sickle-cell anemia without marked borders in the soft tissues of 144. A patient experienced a sudden his thigh. On the tissue bioptate the formation temperature rise up to 39°C. After 6 hours looks like flesh of fish and consists of the temperature normalized. On the 2-nd immature fibroblast-like cells with multiple day the attack recurred: in the period of mitosis growing through the muscles. What paroxysm the temperature reached 41°C, is the most likely diagnosis? apyrexial period began after 8 hours. What A. Fibrosarcoma type of temperature profile is it? B. Fibroma A. Recurrent C. Cancer B. Septic D. Myosarcoma C. Continued E. Myoma D. lntermitting 150. Analysis of amniotic fluid that was E. Hectic obtained as a result of amniocentesis 145. Albinos can't stand sun impact - they (puncture of amniotic sac) revealed cells don't aquire sun-tan but get sunburns. Disturbed the nuclei of which contain sex chromatin metabolism of what aminoacid (Barr's body). What can it be evidence of? underlies this phenomenon? A, Polyploidy A. Glutamic acid B. Trisomy B. Histidine C. Genetic disorders of fetus development C. Tryptophan D. Development of female fetus D. Phenilalanine E. Development of male fetus E. Methionine 151. A 50-year-old patient with typhoid 146. A child has got galactosemia. fever was treated with Levomycetin, Concentration of glucose, in blood has not next day his condition became worse, considerably changed. Deficiency of what temperature rose to 39,6°C. What caused enzyme caused this illness? the complication? A. Galactose-l-phosphate uridyltransferase A. The effect of endotoxin agent B. Hexokinase B. Reinfection C. Galactokinase C. Secondary infection addition D. Amylo-1.6-g1ucosidase D. Irresponsiveness of an agent to the E. Phosphoglucomutase levomycetin 147. A. patient with suspected diphtheria E. Allergic reaction went through bacterioscopic examination. 152. A patient with infectious mononucleosis Examination of throat swab revealed rodshaped had been taking glucocorticoids for two bacteria with volutin granules. weeks. He was brought into remission, but What etiotropic preparation should he fell ill with acute attack of chronic tonsillitis. be chosen in this case? What action of glucocorticoids caused A. Bacteriophage this complication? B. Interferon A. Antiallergic C. Antidiphtheric antitoxic scrum B. Immunosuppressive D. Diphtheria antitoxin C. Anti-inflammatory E. Eubiotic D. Antishock 148. A young man felt acute pain in the back E. Antitoxic during active drawing up on the horizontal 153. A 40-year-old woman was admitted to
  • 16. the infectious diseases department with high involvement of intestines, liver, lungs. What body temperature. Objectively: marked drug should be prescribed? meningeal symptoms. A spinal cord punction A. Enteroseptol was made. What anatomic formation was B. Tetracycline puncturated? C. Quiniofone A. Carum trigeminale D. Quingamine B. Spatium subarachnoideum E. Metronidasol C. Cisterna cerebellomedularis posterior 159. Tuberculosis can be treated by means D. Spatium subdurale of combined chemotherapy that includes E. Spatium epidurale substances with different mechanisms of 154. A patient has got a spasm of smooth action. What antituberculous medication muscles of bronchi. Activators of what inhibits transcription of RNA into DNA in membrane cytoreceptors are phisiologically mycobacteria? reasoned to stop an attack? A. Isoniazid A. β-adrenoreceptors B. Para-aminosalicylic acid B. M-cholinoreccptors C. Rifampicin C. α- and β-adrenoreceptors D. Ethionamide D. H-cholinoreceptors E. Streptomycin E. α -adrenoreceptors 160. A patient who was previously ill with 155. A patient had been taking glucocorticoids for mastectomy as a result of breast cancer was a long time. When the preparation prescribed radiation therapy. What vitamin was withdrawn he developed the symptoms preparation has marked radioprotective of disease aggravation, decreased blood action caused by antioxidant activity? pressure and weakness. What is the reason A. Ergocalciferol of this condition? B. Tocopherol acetate A. Appearance of adrenal insufficiency C. Thiamine chloride B. Hyperproduction of ACTH D. Ribof1avin C. Cumulation E. Folic acid D. Sensibilization 161. Inhibition of α-motoneuron of the E. Habituation extensor muscles was noticed after stimulation 156. The energy inputs of a healthy man of α-motoneuron of the flexor have been measured. In what state was the muscles during the experiment on the spinal patient if his energy inputs were less than cord. What type of inhibition is this process the main exchange? based upon? A. Rest A. Lateral B. Nervous exertion B. Recurrent C. Easy work C. Thiamine chloride D. Sleep D. Presynaptic E. Calmness E. Reciprocal 157. An 18-year-old patient came to the 162. While shifting the gaze to the closely outpatient department with the complaints of situated object the refracting power of eye's bleeding trauma in the vestibule of his nose. optical mediums will increase by 10 diopters. On examination: the mechanical injury of It results from changing of such eye the mucous layer of the vestibule without structure: continuation into nasal cavity proper. What A. Vitreous body is the boundary between the vestibule and B. Liquid of the anterior chamber of eye nasal cavity proper? C Cornea A. Nasal limen D. Lens B. Nasal roller E. Muscle that dilatates pupil C. Nostrils 163. In course of metabolic process active D. Nasal septa forms of oxygen including superoxide anion E. Choanes radical are formed in the human body. By 158. A 52-year-old patient has the following means of what enzyme is this anion inactivated? diagnosis: systemic amebiasis with A. Peroxidase
  • 17. B. Superoxide disffiutase ventricular arrhythmias was admitted to C. Catalase the hospital. The patient is taking timolol D. Glutathioneperoxidase drops for glaucoma, daily insulin injections E. Glutathionereductase for diabetes mellitus, and an ACE inhibitor 164. A student is writing a thorough for hypertension. You have decided to use summary of a lecture. Quality of summarizing phenytoin instead of procainamide. What is has considerably worsened when his neighbors the reason? started talking. What kind of inhibition A. The local anesthetic effect of procainamide in the cerebral cortex caused this effect? would potentiate diabetes A. Protective B. The cholinergic effects of procainamide B. Fading would aggravate the diabetes C. Differentiated C. The hypertensive effects of procainarmide D. Delayed would aggravate the hypertension E. External D. The anticholinergic effect of procainamide 165. Analeptical remedy of ret1ective type would aggravate glaucoma from the H -cholinomimetics group was given E. The local anesthetic effect of procainamide to the patient for restoration of breathing would aggravate the hypertension after poisoning with carbon monoxide. 169. Tuberculine was introduced intracutaneously What medicine was prescribed to the patient? to the child for tuberculin test. Marked A. Lobeline hydrochloride hyperemia, B. Mesaton tissue infiltration developed on C. Adrenaline hydrochloride the place of injection in 24 hours. What D. Atropine sulphate mechanism caused these modifications? E. Pentamin A. Immunocomplex cytotoxity 166. It is planned to use the territory of an B. Antibody cytotoxity old cattle burial ground (which is not used C. Granuloma formation for more than 50 years) for building houses. D. Reagin type cytotoxity But ground analysis revealed presence of E. Cell cytotoxity the pathogen of a very dangerous illness. 170. Concentration of pyruvate is increased Which of the indicated microorganisms is likely in the patient's blood, the most of which is to remain in the ground for such a long excreted with urine. What avitaminosis has time? the patient? A. Brucella abortus A. Avitaminosis B6 B. Mycobacteriurn bovis B. Avitaminosis E C. Francisella tubarensis C. Avitaminosis B2 D. Bacillus anthracis D. Avitaminosis B1 E. Yersinia pestis E. Avitaminosis B3 167. A 22-year-old patient was admitted 171. Upper neck node or Sympathetic trunk to the hospital with Complaints of heavy was removed from the rabbit on experiment. nasal breathing. During the examination Reddening and increased temperature of of her nasal cavity the doctors found thickened the skin of head is observed. What disorder mucous membrane, a lot of mucus of peripheral circulation of the blood has and nodular infiltrates without erosions in developed? the nose. The nasal rhinosc1eroma was A. Neurotonic arterial hyperemia diagnosed. B. Neuroparalytic arterial hyperemia The biopsy was taken. What typical C. Stasis morphological D. Venous hyperemia changes may be found? E. Metabolic arterial hyperemia A. Granulomas with Larnghan's cells 172. Diagnostic scraping was performed B. Granulomas with foreign bady cells to the woman with dysfunctional uterine C. Interstitial inflammation bleeding. Mu1tip1e convoluted glands, D. Granulomas with Virchow`s cells ganglially dilated lumens of some glands E. Granulomas with Mikuliez's cel1s were revealed histologically in the scrape. 168 A 55-year-old patient with continuing Name the type or general pathological
  • 18. process in endometry: notes that he needed to buy bigger shoes A. Hypertrophic excrescence three times. What is the main reason of such B. Metaplasia disproportional enlargement of different C. Displasia parts of the body? D. Hyperplasia glandulocystica A. Joints dystrophy development E. Atrophy B. Increased sensitivity of the tissues to 173. While enrolling a child to school growth hormone Mantu`s test was made to define whether C. Joints chronic inflammation development revaccination was needed. The test result is D. Increased sensitivity of the tissues to negative. What does this test result mean? insulin A. Absence of antitoxic immunity to the E. Cartilaginous tissue proliferation under tuberculosis growth hormone influence B. Presence of cell immunity to the 178. A 54-vear-old man was admitted to the tuberculosis hospital with complaints of pain in the right C. Absence of cell immunity to the subcostal region, vomiting with blood. tuberculosis Objectively: enlarged liver, varicose veins in D. Presence of antibodies for tubercle bacillus the stomach and esophagus. Disfunction of E. Absence of antibodies for tubercle bacillus what vessel is likely to have taken place? 174. Buffer capacity of a worker's blood was A. Vena cava superior decreased due to exhausting muscular work. B. Vena porta By coming of what acid substance in the C. Vena hepatica blood can this state be explained? D. /ena cava inferior A. α-ketoglutarate E. Aorta abdominalis B. Lactate 179. Under SOme diseases it iS observed C. 1,3-bisphosphoglycerate aldosteronism accompanied by hypertension D.3-phosphoglycerate and edema due to sodium retention in E. Pyruvate the organism. What organ of the internal 175. A man who was bitten by the unknown secretion is affected under aldosteronism? dog applied to the surgeon. Wide ragged A. Testicle woundes were localized on the face. What B. Pancreas curative-prophylactic aid should be given to C. Ovaries prevent rabies? D. Adrenal glands A. Prescribe combined antibiotic therapy E. Hypophysis B. Hospitalize the patient and keep under 180. Microscopic examination of the sputum the doctor`s of a patient with pneumonia occasionally C. Start immunisation with rabies vaccine revealed some larvae. Eosinophiles were D. Immediately inject normal gammaglobulin detected on blood examination. What E. Immediate injection of DPT(Diphtheria, helminthiasis can be diagnosed? Pertusis, Tetanus) vaccine A. Enterobiosis 176. A 25-year-old patient complained of B. Opisthorchosis the decreased vision. Accommodation disorders, C. Ascariasis dilated pupil, lack of reaction for D. Trichocephaliasis the light were revealed on examination. E. Paragonimiasis What muscles function is disturbed? 181. A duodenal content smear of a patient A. Pupil dilating muscle, ciliary with indigestion contains protosoa 10-18 B. Pupil narrowing muscle, ciliary mcm large. They have piriform bodies, 4 pairs C. Pupil narrowing and dilating muscles of filaments, two symmetrically located D. Inferior oblique muscle, ciliary nuclei in the broadened part of body. What E. Lateral rectus muscle, pupil narrowing kind of the lowest organisms is it? 177. A 46-year-old patient has complained A. Larnblia of headache, fatigue, thirst, pains in the spine and B. Dysentery ameba joints for the last 2 years. Clinically C. Trichomonas observed disproportional enlargement of D. Intestinal ameba hands: feet, nose, superciliary arches. He E. Balantidium
  • 19. 182. RNA-polymerase B(II) is blocked due A. Glutamic to amanitine poisoning (poison of deathcup). B. Tryptophan It disturbs: C. Tyrosine A. Synthesis of t-RNA D. Valine B. Synthesis of m-RNA E. Lisine C. Maturation of m-RNA 188. Microscopic examination of the D. Primers synthesis enlarged neck gland of a 14-year-old girl E. Reverse transcription revealed destruction of the tissue structure 183. As a result of craniocerebral trauma of the node, absence of the lymph folli- a patient reveals the following symptoms: cles, sclerotic areas and necrosis foci, cell intention tremor, dysmetry, adiadochokinesis, constitution of the node is polymorphous, dysarthria. What structure of the brain lymphocites. eosinophiles, big atypical is injured? cells with multilobular nuclei (Beresovsky- A. Black substance Sternberg cells) and mononuclear cells of B. Motor cortex the large size are present. What is the most C. Striatum likely diagnosis? D. Pale'sphere A. Chronic lympholeucosis B. E. Cerebellum Lymphogranulomatosis 184. Moving of the daughter chromatids to C. Berkitt's lymphoma the poles of the cell is observed in the mitotically D. Acute lympholeucosis dividing cell. At what stage of the E. Fungoid mycosis mitotic cycle is this cell? 189. An isolated muscle of a frog is rhythmi- A. Interphase cally irritated with electric impulses. Every B. Prophase next impulse is in a period of relaxation from C. Telophase the previus contraction. What contraction of the D. Metaphase muscle occurs? Ё. Anaphase A. Single 185. Autopsy of a 12-year-old girl revealed: B. Continuous (smooth) tetanus multiple cutaneous hemorrhages (mostly C. Asynchronous into the skin of buttocks, lower extremi- D. Tonic ties), serous and mucous membrane E. Waved tetanus hemorrhages, cerebral hemorrhages. 190. On autopsy it was revealed: pia mater Adrenal glands show focal necrosis and of the upper parts of cerebral hemisphere massive hemorrhages; kidneys show is plethoric, of yellowish-green color, soaked necrotic nephrosis, suppurative arthritis, with purulent and fibrose exudate, looks like iridocyclitis, vasculitis. What is the most a cap. What disease is it typical for? probable diagnosis'? A. Grippal meningitis A. Epidemic typhus B. Tuberculous meningitis B. Systemic lupus erythematosus С Meningitis connected with typhus С. Radiation sickness D. Meningococcal meningitis D. Periarteritis nodosa E. Meningitis connected with anthrax E. Meningococcemia 191. A 50-year-old man has felt vague 186. A patient working at a chemical abdominal discomfort within past 4 plant was admitted to the toxicological months. Physical examination revealed department with mercury poisoning. What no lymphadenopathy, and no abdominal medicine should be used? masses or organomegaly at palpation. Bowel A. Isonitrozin sounds are heard. An abdominal CT scan B. Naloxone shows a 20 cm retroperitoneal soft tissue C. Enterosorbent mass obscuring the left psoas muscle. A D. Unithiol stool specimen tested for occult blood is E. Activated carbon negative. Which of the following neoplasm’s 187. Cerebral trauma caused increase of ammonia is this man most likely to have? formation. What aminoacid takes part in removal A. Melanoma of ammonia from cerebral tissue? B. Hamartoma C. Lipoma
  • 20. D. Lymphoma 197. A woman who was infected with E. Adenocarcinoma toxoplasmosis during the pregnancy has 192. A child has inhaled a button. Where is born a child with multiple congenital it likely to be? defects. This is a result of: A. In the esophagus A. Recombination B. In the left main bronchus B. Cancerogenesis C. In the larynx C. Biological mutogenesis D. In the right main bronchus D. Chemical mutogenesis E. In the trachea E. Teratogencsis 193. A 68-year-old woman can't move her 198. After a trauma a 44-year-old patient upper and lower right extremities due to had a rupture of left palm muscle tendons the stroke. Muscle tone of these extremities and and of the superficial blood vessels. After their reflexes are increased. There are operation and removal of the most part of pathological reflexes. What form of paralysis the necrotically changed muscle tissue the is it? bloodstream was normalized. What vessels A. Monoplegia have helped to restore the bloodstream? B. Paraplegia A. Aa. metacarpeae palmares C. Tetraplegia B. Aa.digitales palmares communes D. Dissociation C. Arcus palmaris superficialis E. Hemiplegia D. Aa.perforantes 194. During the fetal period of the E. Arcus palmaris profundus development in the vascular system of 199. A patient who has been strictly keeping the fetus a large arterial (Botallo's) to a certain diet for 10 days went through duct is functioning which converts into examination of respiratory coefficient. It Hg.arteriosum after birth. What anatomical was determined that it was equal 1. What formations does this duct connect? have the patient been keeping to? A. Aorta and superior vena cava A. With domination of proteins and fat B. Aorta and inferior vena cava B. With domination of fat and carbohydrates C. Pulmonary trunk and aorta C. With domination of carbohydrates D. Right and left auricles D. Mixed E. Pulmonary trunk and superior vena cava E. With domination of proteins and 195. An intraoperational biopsy of mammal carbohydrates gland has revealed the signs of atypical tissue 200. A patient has tissue ischemia below the presented by disorder of parenchyma knee joint accompanied with intermittent stroma proportion with domination of the claudication. What artery occlusion should last, gland structures of different size and be suspected? shape, lined with single-layered proliferative A. Popliteal artery epithelium. What is the most probable B. Anterior tibial artery diagnosis? C. Proximal part of femoral artery A. Noninfiltrative cancer D. Posterior tibial artery B. Infiltrative cancer E. Peroneal artery C. Papilloma D. Fibroadenoma E. Mastitis 196. A person was selling "homemade pork" sausages at the market. State sanitary inspector suspected falcification of the sausages. What serological immune reaction can identify food substance? A. Agglutination test B. Immunofluorescence test C. Precipitation test D. Indirect hemagglutination test E. Complement-fixation test